History of Mathematics Prelim

You might also like

Download as pdf or txt
Download as pdf or txt
You are on page 1of 89

Davao Vision Colleges Inc.

Stone Rock Village Catalunan Grande, Davao City


Tel. No. (082) 295-6696 / 297-4750 (Fax)

History of Mathematics
(MATH 100)
Prelim Modules
S.Y 2022-2023

Name of the Student:____________________________


Address:______________________________________
Contact Number:_______________________________

Teacher: Ferdinand M. Serdiña, LPT

fserdina@dvci-edu.com 0907-012-5620 Sandro Serdiña

Checked by: Noted by:


Marilou E, Tejero, MAEd Rosie C. Dela Cruz, Ph. D
SHS Principal HR / Interim President
Egyptian Mathematics

Our Þrst knowledge of mankind’s use of mathematics beyond mere


counting comes from the Egyptians and Babylonians. Both civi-
lizations developed mathematics that was similar in some ways but
different in others. The mathematics of Egypt, at least what is
known from the papyri, can essentially be called applied arithmetic.
It was practical information communicated via example on how to
solve speciÞc problems.

Pyramids at Giza

This point, that mathematics was communicated by example, rather


than by principle, is signiÞcant and is different than today’s mathe-
matics that is communicated essentially by principle with examples
to illustrate principles. The reasons for this are unknown but could
be due partly to the fact that symbolism, the medium of principles,
did not exist in these early times. Indeed, much of mathematics for
many centuries was communicated in this way. It may be much eas-
ier to explain to a young student an algorithm to solve a problem
and for them to learn to solve like problems, than to explain the
abstract concept Þrst and basing examples upon this concept.

1 Basic facts about ancient Egypt.

Egyptian hieroglyphics are in great abundance throughout Egypt.


They were essentially indecipherable until 1799 when in Alexandria
the trilingual Rosetta Stone was discovered. The Rosetta stone,
an irregularly shaped tablet of black basalt measuring about 3 feet
Egypt 2

9 inches by 2 feet 4 inches, was found near the town of Rosetta


(Rashid) just a few miles northwest of Alexandria. Written in the
two languages (Greek and Egyptian but three writing systems (hi-
eroglyphics, its cursive form demotic script, and Greek, it provided
the key toward the deciphering of hieroglyphic writing. The inscrip-
tions on it were the benefactions conferred by Ptolemy V Epiphanes
(205 - 180 BCE) were written by the priests of Memphis. The trans-
lation was primarily due to Thomas Young1 (1773 - 1829) and

Temple at Al Karnak

Jean Francois Champollion (1790-1832)2 (1790-1832), who, very


early in his life was inspired to Egyptology by the French math-
ematician Jean Baptiste Joseph Fourier (1768 - 1830). Champollion
completed the work begun by Young and correctly deciphered the
complete stone. An Egyptologist of the Þrst rank, he was the Þrst to
recognize the signs could be alphabetic, syllabic, or determinative
(i.e. standing for complete ideas) He also established the original
language of the Rosetta stone was Greek, and that the hieroglyphic
text was a translation from the Greek. An unusual aspect of hiero-
glyphics is that they can be
1 English physician and physicist established the principle of interference of light and thus

resurrected the century-old wave theory of light.


2 French historian and linguist who founded scientiÞc Egyptology. Academically prodigious,

he had already mastered six ancient Oriental languages by the age of 16. At 19, he was
appointed professor of history at the lycé of Grenoble, where he was to remain for eight years.
Deciphering hieroglyphics became his constant preoccupation
Egypt 3

read from left to right, or


right to left, or vertically
(top to bottom). It is the
orientation of the glyphs that
gives the clue; the direction
of people and animals face
toward the beginning of the
line.
For the Egyptians
writing was an esthetic
experience, and they viewed
their writing signs as “God’s
words.” This could explain
the unnecessary complexity,
in face of the fact that
obviously simpliÞcations
would certainly have
occurred if writing were
designed for all citizens. Rosetta Stone

The demotic script was for more general use, the hieroglyphics con-
tinued to be used for priestly and formal applications.
The Egyptians established an annual calendar of 12 months of
30 days each plus Þve feast days. Religion was a central feature of
Egyptian society. There was a preoccupation with death. Many
of Egypt’s greatest monuments were tombs constructed at great
expense, and which required detailed logistical calculations and at
least basic geometry.
Construction projects on a massive scale were routinely carried
out. The logistics of construction require all sorts of mathematics.
You will see several mensuration (measurement) problems, simple
algebra problems, and the methods for computation.
Our sources of Egyptian mathematics are scarce. Indeed, much of
our knowledge of ancient Egyptian mathematics comes not from the
hieroglyphics3 (carved sacred letters or sacred letters) inscribed on
the hundreds of temples but from two papyri containing collections
3 The words “hieroglyph” or “hieroglyphic” are derived from the Greek words grammata

hiera or grammata hieroglyphika respectively.


Egypt 4

of mathematical problems with their solutions.

• The Rhind Mathematical Papyrus named for A.H. Rhind (1833-


1863) who purchased it at Luxor in 1858. Origin: 1650 BCE
but it was written very much earlier. It is 18 feet long and
13 inches wide. It is also called the Ahmes Papyrus after the
scribe that last copied it.
• The Moscow Mathematical Papyrus purchased by V. S. Golen-
ishchev (d. 1947). Origin: 1700 BC. It is 15 ft long and 3 inches
wide. Two sections of this chapter offer highlights from these
papyri.

Papyrus, the writing material of ancient times, takes its name from
the plant from which it is made. Long-
cultivated in the Nile delta
region in Egypt, the
Cyperus papyrus was grown
for its stalk, whose inner
pith was cut into thin strips
and laid at right angles on
top of each other. When
pasted and pressed
together, the result was
smooth, thin, cream-colored
papery sheets, normally
about Þve to six inches
wide. To write on it
brushes or styli, reeds with
crushed tips, were dipped
into ink or colored liquid. From the Duke Papyrus Archive*
A remarkable number of papyri, some dating from 2,500 BCE,
have been found, protected from decomposition by the dry heat
of the region though they often lay unprotected in desert sands or
burial tombs.
* See the URL: http://odyssey.lib.duke.edu/papyrus/texts/homepage.html
Egypt 5

2 Counting and Arithmetic — basics

The Egyptian counting system was decimal. Though non positional,


it could deal with numbers of great scale. Yet, there is no apparent
way to construct numbers arbitrarily large. (Compare that with
modern systems, which is positional, which by its nature allows and
economy for expressing huge numbers.)
The number system was decimal with special symbols for 1, 10,
100, 1,000, 10,000, 100,000, and 1,000,000. Addition was accom-
plished by grouping and regrouping. Multiplication and division
were essentially based on binary multiples. Fractions were ubiqui-
tous but only unit fractions, with two exceptions, were allowed. All
other fractions were required to be written as a sum of unit fractions.
Geometry was limited to areas, volumes, and similarity. Curiously,
though, volume measures for the fractional portions of the hekat
a volume measuring about 4.8 liters, were symbolically expressed
differently from others.
Simple algebraic equations were solvable, even systems of equa-
tions in two dimensions could be solved.
Egypt 6

Symbolic notation for numbers.

1 = vertical stroke
10 = heal bone
100 = a snare
1,000 = lotus ßower
10,000 = a bent Þnger
100,000 = a burbot Þsh
1,000,000 = a kneeling Þgure

Note though that there are numerous interpretations of what


these hieroglyphs might represent.

Numbers are formed by grouping.


Egypt 7

Addition is formed by grouping

Note alternate forms for these numbers.

Multiplication is basically binary.


Example: Multiply: 47 × 24

47 × 24
47 1 doubling process
94 2
188 4
376 8 *
752 16 *

Selecting 8 and 16 (i.e. 8 + 16 = 24), we have


24 = 16 + 8
47 × 24 = 47 × (16 + 8)
= 752 + 376
= 1128

Division is also basically binary.


Example: Divide: 329 ÷ 12
Egypt 8

329 ÷ 12
12 1 doubling 329
24 2 -192
48 4 137
96 8 -96
192 16 41
384 32 -24
17
-12
5

Now
329 = 16 × 12 + 8 × 12 + 2 × 12 + 1 × 12 + 5
= (16 + 8 + 2 + 1) × 12 + 5
So,
5 1 1
329 ÷ 12 = 27 = 27 + + .
12 3 12
Obviously, the distributive laws for multiplication and division were
well understood.

Fractions It seems that the Egyptians allowed only unit fractions,


with just two exceptions, 23 and 34 . All other fractions must be
converted to unit fractions. The symbol for unit fractions was
a ßattened oval above the denominator. In fact, this oval was
the sign used by the Egyptians for the “mouth.” In the case of
the volume measure hekat, the commonly used fractional parts of
1 1 1
, , , 1 , 1 ,and 64
2 4 8 16 32
1
, were denoted by parts of the symbol for the
Horus-eye, symbolized as .4 For ordinary fractions, we have the
following.

1/3 1/10 1/25


There were special symbols for the fractions 12 , 2
3
, 3
4
, of which
one each of the forms is shown below.
4 See Ifrah, p 169.
Egypt 9

1/2 2/3 3/4

All other fractions must be converted to unit fractions. For example:


2 1 1
= +
15 10 30

Obviously there are but two cases to consider, unit fractions and
fractions with numerator two. All fractions can be reduced to a
sum of such fractions. Ahmes gives a table of unit fractions decom-
positions for fractions with numerator two.
2
n
1/p + 1/q + 1/r+. . .
5 3 15
7 4 28
9 6 18
11 6 66
13 8 52 104
15 10 30
..
.

• Decompositions are not necessarily unique. The Egyptians did


favor certain fractions and attempt to use them when possible.
For example, they seems to prefer taking halves when possible.
Thus the representation for 2/15 as
2
= 1/30 + 1/10
15
• The exact algorithm for determination for the decomposition is
unknown and this is an active topic of research today. However,
in other papyri, there is some indication of the application of
the formula
2 1 1
= p+q +
p·q p· 2 q · p+q
2
being used. It gives some, but not all, of the table, and certainly
does not give decompositions into three or more fractions.
Egypt 10

• It seems certain that the Egyptians understood general rules


for handling fractions.

3 The Ahmes Papyrus

The Ahmes was written in hieratic, and probably originated from


the Middle Kingdom: 2000-1800 BC. It claims to be a “thorough
study of all things, insight into all that exists, knowledge of all
obscure secrets.” In fact, it is somewhat less. It is a collection of
exercises, substantially rhetorical in form, designed primarily for
students of mathematics. Included are exercises in

• fractions
• notation
• arithmetic
• algebra
• geometry
• mensuration

The practical mathematical tools for construction?

To illustrate the level and scope of Egyptian mathematics of this


period, we select several of the problems and their solutions as found
in the two papyri. For example, beer and bread problems are com-
mon in the Ahmes.
Problem 72. How many loaves of ”strength” 45 are equivalent to
100 loaves of strength 10? Fact:

strength := grain 1density


Egypt 11

Invoking the rule of three5 , which was well known in the ancient
world, we must solve the problem:
x 100
=
45 10

Answer: x = 100/10 × 45 = 450 loaves.

Problem 63. 700 loaves are to be divided among recipients where


the amounts they are to receive are in the continued proportion
2 1 1 1
: : :
3 2 3 4
Solution. Add
2 1 1 1 7
+ + + = .
3 2 3 4 4

700 4
= 700 ·
7/4 7
2 2
= 700( + )
7 7
2 1 2 1
= 700( + + + )
7 7 28 14
1 1
= 700( + )
2 14
= 350 + 50
= 400

The Þrst value is 400. This is the base number. Now multiply
each fraction by 400 to obtain the recipient’s amount. Note the
algorithm nature of this solution. It reveals no principles at all. Only
when converting to modern notation and using modern symbols do
we see that this is correct We have
2 1
x1 3 x2
= 1, = 21 ,
x2 2
x 3 3
5 The rule of three was the rule to determine the fourth and unknown quantity in the

expression ab = dc in which the other three are known


Egypt 12

etc. This will be the case if there is a base number a such that
2
x1 = a
3
1
x2 = a
2
1
x3 = a
3
1
x4 = a
4
Thus
2 1 1 1
x1 + x2 + x3 + x4 = ( + + + )a = 700
3 2 3 4
7
Now add the fractions to get 4 and solve to get
a = 400.
Now compute x1 , x2 , x3 , x4 . This problem seems to indicate a
type of hierarchical chain for the distribution of goods was relatively
common. Similar problems are relatively rare in modern texts.

The solution of linear algebra problems is present in the Ahmes.


Equations of the modern form
x + ax = b or x + ax + bx = x,
where a, b, and c are known are solved. The unknown, x, is called
the heep. Note the rhetorical problem statement.

Problem 24. Find the heep if the heap and a seventh of the heep is
19. (Solve x + x/7 = 19.)

Method. Use the method of false position. Let g be the guess.


Substitute g + ag = c. Now solve c · y = b. Answer: x = g · y. Why?

Solution. Guess g = 7.
7 + 1/7 · 7 = 8
3 1 1
19 ÷ 8 = 2 + =2+ +
8 4 8
Egypt 13

Answer:
3 1 1 1 1
7 · (2 + ) = 7(2 + + ) = 16 + +
8 4 8 2 8

Geometry and Mensuration Most geometry is related to mensura-


tion. The Ahmes contains problems for the areas of

• isosceles triangles (correct)


• isosceles trapezoids (correct)
• quadrilaterals (incorrect)
• frustum (correct)
• circle (incorrect)
• curvilinear areas

In one problem the area for the quadrilateral was given by


b1 + b2 h1 + h2
A=( )( )
2 2
which of course is wrong in general but correct for rectangles. Yet
the “Rope stretchers” of ancient Egypt, that is the land surveyors,
often had to deal with irregular quadrilaterals when measuring ar-
eas of land. This formula is quite accurate if the quadrilateral in
question is nearly a rectangle.
Egypt 14

b2
h2

h1

b1

Quadrilateral

The area for the triangle was given by replacement b2 = 0 in the


quadrilateral formula
b1 h1 + h2
A=( )( )
2 2

h
2
h1

b1

Triangle

On Rigor. There is in Egyptian mathematics a search for relation-


ships, but the Egyptians had only a vague distinction between the
exact and the approximate. Formulas were not evident. Only solu-
tions to speciÞc problems were given, from which the student was
left to generalize to other circumstances. Yet, as we shall see, sev-
eral of the great Greek mathematicians, Pythagoras , Thales, and
Eudoxus to name three, studied in Egypt. There must have been
more there than student exercises to learn!

Problem 79. This problem cites only “seven houses, 49 cats, 343
mice, 2401 ears of spelt, 16,807 hekats.”
Egypt 15

Note the similarity to our familiar nursery rhyme:

As I was going to St. Ives,


I met a man with seven wives;
Every wife had seven sacks,
Every sack had seven cats,
Every cat had seven kits.
Kits, cats, sacks, and wives,
How many were going to St. Ives?

This rhyme asked for the very impractical sum of all and thus il-
lustrates some knowledge and application of geometric progressions.

Problem 50. A circular Þeld of diameter 9 has the same area as a


square of side 8. This gives an effective π = 3 16 .

Problem 48 gives a hint of how this formula is constructed.

Side length = 9

Trisect each side. Remove the corner triangles. The resulting oc-
tagonal Þgure approximates the circle. The area of the octagonal
Þgure is:
1
9 × 9 − 4( · 3 · 3) = 63 ≈ 64 = 82
2
Egypt 16

Thus the number


8 13
4( )2 = 3
9 81
plays the role of π. That this octagonal Þgure, whose area is easily
calculated, so accurately approximates the area of the circle is just
plain good luck. Obtaining a better approximation to the area using
Þner divisions of a square and a similar argument is not simple.

Geometry and Mensuration

Problem 56 indicates an understanding of the idea of geometric


similarity. This problem discusses the ratio
rise
run
The problem essentially asks to compute the cot α for some angle
α. Such a formula would be need for building pyramids.

Rise

Run

Note the obvious application to the construction of a pyramid for


which the formula for the volume, V = 13 b2 h, was known. (How did
they Þnd that?)
Egypt 17

b
Pyramid

Geometry and Mensuration

The are numerous myths about the presumed geometric relationship


among the dimensions of the Great Pyramid. Here’s one:

[perimeter of base]=
[circumference of a circle of radius=height]

Such a formula would yield an effective π = 3 17 , not π = 3 16 , as


already discussed.

4 The Moscow Papyrus

The Moscow papyrus contains only about 25, mostly practical, ex-
amples. The author is unknown. It was purchased by V. S. Golen-
ishchev (d. 1947) and sold to the Moscow Museum of Fine Art.
Origin: 1700 BC. It is 15 feet long and about 3 inches wide.
Egypt 18

Problem 14. Volume of a frustum. The scribe directs one to square


the numbers two and four and to add to the sum of these squares
the product of two and four. Multiply this by one third of six. “See,
it is 56; your have found it correctly.” What the student has been
directed to compute is the number
1
V = · 6(22 + 42 + 2 · 4) = 56
3
Here’s the picture that is found in the Moscow Papyrus.
Egypt 19

Here’s the modern version of the picture and a perspective drawing.

56 6

Frustum

4 2
2

The general formula for a frustum was evidently known to the


Egyptians. It is:
1
V = h(b21 + b1 b2 + b22 )
3
Taking b1 = 0, we get the formula
1
V = hb2
3
This was evidently known also.

Question. Speculate on how the Egyptians could have known the


formula for a frustum, given that its derivation depends on the meth-
ods of modern calculus.
Egypt 20

Problem 10 Compute the surface area of a Quonset type hut roof,


which is the earliest estimation of curvilinear area.

Quonset roof

5 Summary of Egyptian Mathematics

In the few bullet items below we give a summary of known Egyptian


mathematical achievements. Records of conquests of pharohs and
other facts of Egyptian life are in abundance throughout Egypt, but
of her mathematics only traces have been found. These fragments,
from a civilization that lasted a millennium longer than the entire
Christian era, that undertook constructions projects on a seen not
seen again6 until this century, and that created abundance from a
desert, allow only the following conclusions.

• Egyptian mathematics remained remarkably uniform through-


out time.
• It was built around addition.
• Little theoretical contributions were evident. Only the slightest
of abstraction is evident. Yet exact versions of difficult to Þnd
formulas were available.
• It was substantially practical. The texts were for students. No
“principles” are evident, neither are there laws, theorems, ax-
ioms postulates or demonstrations; the problems of the papyri
are examples from which the student would generalize to the
6 excluding the Great Wall of China constructed in third century BCE
Egypt 21

actual problem at hand. The papyri were probably not written


for self-study. No doubt there was a teacher present to assist
the student learning the examples and then giving “exercises”
for the student to solve.
• There seems to be no clear differentiation between the concepts
of exactness and approximate.
• Elementary congruencies were used only for mensuration.

Yet, there must have been much more to Egyptian mathemat-


ics. We know that Thales, Pythagoras and others visited Egypt to
study. If there were only applied arithmetic methods as we have
seen in the papyri, the trip would have had little value. But where
are the records of achievement? Very likely, the mathematics ex-
tant was absorbed into the body of Greek mathematics – in an age
where new and better works completely displaced the old, and in
this case the old works written in hieroglyphics. Additionally, the
Alexandrian library, one place where ancient Egyptian mathemat-
ical works may have been preserved, was destroyed by about 400
CE.
Babylonian Mathematics1

1 Introduction

Our first knowledge of mankind’s use of mathematics comes from the


Egyptians and Babylonians. Both civilizations developed mathematics
that was similar in scope but different in particulars. There can be no
denying the fact that the totality of their mathematics was profoundly
elementary2 , but their astronomy of later times did achieve a level com-
parable to the Greeks.

Assyria

2 Basic Facts

The Babylonian civilization has its roots dating to 4000BCE with the
Sumerians in Mesopotamia. Yet little is known about the Sumerians.
Sumer was first settled between 4500 and 4000 BC by a non-Semitic
1 °2002,
c G. Donald Allen
2 Neugebauer, 1951
Babylonian Mathematics 2

people who did not speak the Sumerian language. These people now
are called Ubaidians, for the village Al-Ubaid, where their remains were
first uncovered. Even less is known about their mathematics. Of the
little that is known, the Sumerians of the Mesopotamian valley built
homes and temples and decorated them with artistic pottery and mo-
saics in geometric patterns. The Ubaidians were the first civilizing force
in the region. They drained marshes for agriculture, developed trade
and established industries including weaving, leatherwork, metalwork,
masonry, and pottery. The people called Sumerians, whose language
prevailed in the territory, probably came from around Anatolia, proba-
bly arriving in Sumer about 3300 BC. For a brief chronological outline
of Mesopotamia see
http://www.gatewaystobabylon.com/introduction/briefchonology.htm. See
also http://www.wsu.edu:8080/˜dee/MESO/TIMELINE.HTM for more
detailed information.
The early Sumerians did have writing for numbers as shown below.
Owing to the scarcity of resources, the Sumerians adapted the ubiquitous
clay in the region developing a writing that required the use of a stylus
to carve into a soft clay tablet. It predated the

1 10 60 600 3,600 36,000

cuneiform (wedge) pattern of writing that the Sumerians had developed


during the fourth millennium. It probably antedates the Egyptian hiero-
glyphic may have been the earliest form of written communication. The
Babylonians, and other cultures including the Assyrians, and Hittites,
inherited Sumerian law and literature and importantly their style of writ-
ing. Here we focus on the later period of the Mesopotamian civilization
which engulfed the Sumerian civilization. The Mesopotamian civiliza-
tions are often called Babylonian, though this is not correct. Actually,
Babylon3 was not the first great city, though the whole civilization is
called Babylonian. Babylon, even during its existence, was not always
3 The first reference to the Babylon site of a temple occurs in about 2200 BCE. The name means “gate

of God.” It became an independent city-state in 1894 BCE and Babylonia was the surrounding area. Its
location is about 56 miles south of modern Baghdad.
Babylonian Mathematics 3

the center of Mesopotamian culture. The region, at least that between


the two rivers, the Tigris and the Euphrates, is also called Chaldea.
The dates of the Mesopotamian civilizations date from 2000-600
BCE. Somewhat earlier we see the unification of local principates by
powerful leaders — not unlike that in China. One of the most powerful
was Sargon the Great (ca. 2276-2221 BC). Under his rule the region
was forged into an empire called the dynasty of Akkad and the Akka-
dian language began to replace Sumerian. Vast public works, such as
irrigation canals and embankment fortifications, were completed about
this time. These were needed because of the nature of the geography
combined with the need to feed a large population. Because the Trigris
and Euphrates would flood in heavy rains and the clay soil was not very
absorptive, such constructions were necessary if a large civilization was
to flourish.
Later in about 2218 BCE tribesmen from the eastern hills, the
Gutians, overthrew Akkadian rule giving rise to the 3rd Dynasty of Ur.
They ruled much of Mesopotamia. However, this dynasty was soon
to perish by the influx of Elamites from the north, which eventually
destroyed the city of Ur in about 2000 BC. These tribes took command
of all the ancient cities and mixed with the local people. No city gained
overall control until Hammurabi of Babylon (reigned about 1792-1750
BCE) united the country for a few years toward the end of his reign.
The Babylonian “texts” come to us in the form of clay tablets,
usually about the size of a hand. They were inscribed in cuneiform, a
wedge-shaped writing owing its appearance to the stylus that was used
to make it. Two types of mathematical tablets are generally found,
table-texts and problem texts. Table-texts are just that, tables of values
for some purpose, such as multiplication tables, weights and measures
tables, reciprocal tables, and the like. Many of the table texts are clearly
“school texts”, written by apprentice scribes. The second class of tablets
are concerned with the solutions or methods of solution to algebraic or
geometrical problems. Some tables contain up to two hundred problems,
of gradual increasing difficulty. No doubt, the role of the teacher was
significant.
Babylon fell to Cyrus of Persia in 538 BC, but the city was spared.
Babylonian Mathematics 4

The Darius inscription on cliff near Bisotun

The great empire was finished. However, another period of Babylonian


mathematical history occurred in about 300BCE, when the Seleucids,
successors of Alexander the Great came into command. The 300 year
period has furnished a great number of astronomical records which
are remarkably mathematical — comparable to Ptolemy’s Almagest.
Mathematical texts though are rare from this period. This points to the
acuity and survival of the mathematical texts from the old-Babylonian
period (about 1800 to 1600 BCE), and it is the old period we will focus
on.

The use of cuneiform script formed a strong bond. Laws, tax ac-
counts, stories, school lessons, personal letters were impressed on soft
clay tablets and then were baked in the hot sun or in ovens. From one re-
gion, the site of ancient Nippur, there have been recovered some 50,000
tablets. Many university libraries have large collections of cuneiform
tablets. The largest collections from the Nippur excavations, for ex-
ample, are to be found at Philadelphia, Jena, and Istanbul. All total,
at least 500,000 tablets have been recovered to date. Even still, it is
estimated that the vast bulk of existing tablets is still buried in the ruins
of ancient cities.
Babylonian Mathematics 5

Deciphering cuneiform succeeded the Egyptian hieroglyphic. In-


deed, just as for hieroglyphics, the key to deciphering was a trilingual
inscription found by a British office, Henry Rawlinson (1810-1895),
stationed as an advisor to the Shah. In 516 BCE Darius the Great, who
reigned in 522-486 BCE, caused a lasting monument4 to his rule to be
engraved in bas relief on a 100 × 150 foot surface on a rock cliff, the
“Mountain of the Gods” at Behistun5 at the foot of the Zagros Moun-
tains in the Kermanshah region of modern Iran along the road between
modern Hamadan (Iran) and Baghdad, near the town of Bisotun. In
antiquity, the name of the village was Bagastâna, which means ‘place
where the gods dwell’.
Like the Rosetta stone, it was inscribed in three languages — Old
Persian, Elamite, and Akkadian (Babylonian). However, all three were
then unknown. Only because Old Persian has only 43 signs and had
been the subject of serious investigation since the beginning of the cen-
tury was the deciphering possible. Progress was very slow. Rawlinson
was able to correctly assign correct values to 246 characters, and more-
over, he discovered that the same sign could stand for different con-
sonantal sounds, depending on the vowel that followed. (polyphony)
It has only been in the 20th century that substantial publications have
appeared. Rawlinson published the completed translation and grammar
in 1846-1851. He was eventually knighted and served in parliament
(1858, 1865-68).
For more details on this inscription, see the article by Jona Lendering
at
http://www.livius.org/be-bm/behistun/behistun01.html. A translation is
included.

3 Babylonian Numbers

In mathematics, the Babylonians (Sumerians) were somewhat more ad-


vanced than the Egyptians.

• Their mathematical notation was positional but sexagesimal.


4 According to some sources, the actual events described in the monument took place be-
tween 522 and 520 BCE.
5 also spelled Bistoun
¯ ¯
Babylonian Mathematics 6

• They used no zero.


• More general fractions, though not all fractions, were admitted.
• They could extract square roots.
• They could solve linear systems.
• They worked with Pythagorean triples.
• They solved cubic equations with the help of tables.
• They studied circular measurement.
• Their geometry was sometimes incorrect.

For enumeration the Babylonians used symbols for 1, 10, 60, 600,
3,600, 36,000, and 216,000, similar to the earlier period. Below are
four of the symbols. They did arithmetic in base 60, sexagesimal.

1 10 60 600
Cuneiform numerals

For our purposes we will use just the first two symbols
∨ = 1 ≺ = 10
All numbers will be formed from these.
Example:
≺≺ ∨ ∨ ∨
= 57
≺≺≺ ∨ ∨ ∨∨
Note the notation was positional and sexagesimal:
≺≺ ≺≺= 20 · 60 + 20
∨ ∨ ∨∨ ≺ ∨ = 2 · 602 + 2 · 60 + 21 = 7, 331

The story is a little more complicated. A few shortcuts or abbre-


viation were allowed, many originating in the Seleucid period. Other
Babylonian Mathematics 7

devices for representing numbers were used. Below see how the number
19 was expressed.

Three ways to express the number 19


= 19 Old Babylonian. The symbol means subtraction.
= 19 Formal
= 19 Cursive form
Seleucid Period(c. 320 BC to c. 620 AD)

The horizontal symbol above the “1” designated subtraction.

There is no clear reason why the Babylonians selected the sexages-


imal system6 . It was possibly selected in the interest of metrology, this
according to Theon of Alexandria, a commentator of the fourth century
A.D.: i.e. the values 2,3,5,10,12,15,20, and 30 all divide 60. Remnants
still exist today with time and angular measurement. However, a num-
ber of theories have been posited for the Babylonians choosing the base
of 60. For example7

1. The number of days, 360, in a year gave rise to the subdivision


of the circle into 360 degrees, and that the chord of one sixth of a
circle is equal to the radius gave rise to a natural division of the
circle into six equal parts. This in turn made 60 a natural unit of
counting. (Moritz Cantor, 1880)
2. The Babylonians used a 12 hour clock, with 60 minute hours.
That is, two of our minutes is one minute for the Babylonians.
(Lehmann-Haupt, 1889) Moreover, the (Mesopotamian) zodiac
was divided into twelve equal sectors of 30 degrees each.
3. The base 60 provided a convenient way to express fractions from a
variety of systems as may be needed in conversion of weights and
measures. In the Egyptian system, we have seen the values 1/1,
1/2, 2/3, 1, 2, . . . , 10. Combining we see the factor of 6 needed
in the denominator of fractions. This with the base 10 gives 60 as
the base of the new system. (Neugebauer, 1927)
4. The number 60 is the product of the number of planets (5 known at
the time) by the number of months in the year, 12. (D. J. Boorstin,
6 Recall, the very early use of the sexagesimal system in China. There may well be a connection.
7 See Georges Ifrah, The Universal History of Numbers, Wiley, New York, 2000.
Babylonian Mathematics 8

1986)
5. The combination of the duodecimal system (base 12) and the base
10 system leads naturally to a base 60 system. Moreover, duodeci-
mal systems have their remnants even today where we count some
commodities such as eggs by the dozen. The English system of
fluid measurement has numerous base twelve values. As we see
in the charts below, the base twelve (base 3, 6?) and base two
graduations are mixed. Similar values exist in the ancient Roman,
Sumerian, and Assyrian measurements.

fluid
teaspoon tablespoon ounce
1 teaspoon = 1 1/3 1/6
1 tablespoon = 3 1 1/2
1 fluid ounce = 6 2 1
1 gill = 24 8 4
1 cup = 48 16 8
1 pint = 96 32 16
1 quart = 192 64 32
1 gallon = 768 256 128
1 firkin = 6912 2304 1152
1 hogshead = 48384 16128 8064

inch foot yard


1 inch = 1 1/12 1/36
1 foot = 12 1 1/3
1 yard = 36 3 1
1 mile = --- 5280 1760

Note that missing in the first column of the liquid/dry measurement


table is the important cooking measure 1/4 cup, which equals 12
teaspoons.
6. The explanations above have the common factor of attempting to
give a plausibility argument based on some particular aspect of
their society. Having witnessed various systems evolve in modern
times, we are tempted to conjecture that a certain arbitrariness may
be at work. To create or impose a number system and make it apply
to an entire civilization must have been the work of a political
system of great power and centralization. (We need only consider
the failed American attempt to go metric beginning in 1971. See,
http://lamar.colostate.edu/ hillger/dates.htm) The decision to adapt
Babylonian Mathematics 9

the base may have been may been made by a ruler with little more
than the advice merchants or generals with some vested need.
Alternatively, with the consolidation of power in Sumeria, there
may have been competing systems of measurement. Perhaps, the
base 60 was chosen as a compromise.

Because of the large base, multiplication was carried out with the
aide of a table. Yet, there is no table of such a magnitude. Instead
there are tables up to 20 and then selected values greater (i.e. 30, 40,
and 50). The practitioner would be expected to decompose the number
into a sum of smaller numbers and use multiplicative distributivity.

A positional fault??? Which is it?


≺ ≺ = 10 · 60 + 10
= 10 · 602 + 10 = 3, 610
10
= 10 +
60
= 20(???)

1. There is no “gap” designator.


2. There is a true floating point — its location is undetermined except
from context.

? The “gap” problem was overcome in the Seleucid period with


the invention of a “zero” as a gap separator.

We use the notation:


d2 d3
d1 ; d2 , d3 , . . . = d1 + + 2 + ···
60 60
The values d1 ; d2 , d3 , d4 , . . . are all integers.
Example
≺ ∨∨ ≺≺
∨ ≺ ∨ ≺
≺ ∨∨ ≺≺
24 51 10
1; 24, 51, 10 = 1 + + 2+ 3
60 60 60
= 1.41421296
Babylonian Mathematics 10

This number was found on the Old Babylonian Tablet


√ (Yale Collection
#7289) and is a very high precision estimate of 2. We will continue
this discussion shortly, conjecturing on how such precision may have
been obtained.


The exact value of 2, to 8 decimal places is = 1.41421356.

Fractions. Generally the only fractions permitted were such as


2 3 5 12
, , , , ...
60 60 60 60
because the sexagesimal expression was known. For example,
1 10
= = ;≺
6 60
1 ∨ ∨ ∨ ≺≺
=; ,
9 ∨ ∨ ∨ ≺≺

Irregular fractions such as 17 , 11


1
, etc were not normally not used.
There are some tablets that remark, “7 does not divide”, or “11 does
not divide”, etc.

A table of all products equal to sixty has been found.

2 30 16 3, 45
3 20 18 3,20
4 15 20 3
5 12 24 2,30
6 10 25 2,25
8 7,30 27 2,13,20
9 6,40 30 2
10 6 32 1;52,30
12 5 36 1,40
15 4 40 1,30
Babylonian Mathematics 11

You can see, for example that


30
8 × 7; 30 = 8 × (7 + ) = 60
60
Note that we did not use the separatrix “;” here. This is because the
table is also used for reciprocals. Thus
1 7 30
= 0; 7, 30 = + 2
8 60 60
Contextual interpretation was critical.
Remark. The corresponding table for our decimal system is shown
below. Included also are the columns with 1 and the base 10. The
product relation and the decimal expansion relations are valid in base
10.

1 10
2 5
5 2
10 1

Two tablets found in 1854 at Senkerah on the Euphrates date from


2000 B.C. They give squares of the numbers up to 59 and cubes up to
32. The Babylonians used the formula
xy = ((x + y)2 − (x − y)2 )/4
to assist in multiplication. Division relied on multiplication, i.e.
x 1
=x·
y y
There apparently was no long division.
The Babylonians knew some approximations of irregular fractions.
1 1
=; 1, 1, 1 =; 0, 59, 0, 59
59 61
However, they do not appear to have noticed infinite periodic expan-
sions.8
8 In the decimal system, the analogous values are 19 = 0.1111 . . . and 1
11
= 0.090909 . . ..
Note the use of the units “0” here but not for the sexagesimal. Why?
Babylonian Mathematics 12

They also seemed to have an elementary knowledge of logarithms.


That is to say there are texts which concern the determination of the
exponents of given numbers.

4 Babylonian Algebra

In Greek mathematics there is a clear distinction between the geometric


and algebraic. Overwhelmingly, the Greeks assumed a geometric posi-
tion wherever possible. Only in the later work of Diophantus do we see
algebraic methods of significance. On the other hand, the Babylonians
assumed just as definitely, an algebraic viewpoint. They allowed opera-
tions that were forbidden in Greek mathematics and even later until the
16th century of our own era. For example, they would freely multiply
areas and lengths, demonstrating that the units were of less importance.
Their methods of designating unknowns, however, does invoke units.
First, mathematical expression was strictly rhetorical, symbolism would
not come for another two millenia with Diophantus, and then not sig-
nificantly until Vieta in the 16th century. For example, the designation
of the unknown was length. The designation of the square of the un-
known was area. In solving linear systems of two dimensions, the
unknowns were length and breadth, and length, breadth, and width for
three dimensions.

Square Roots. Recall the approximation of 2. How did they get it?
There are two possibilities: (1) Applying the method of the mean. (2)
Applying the approximation
√ b
a2 ± b ≈ a ±
2a
Babylonian Mathematics 13

Yale Babylonian Collection

1;24,51,10

30

42;25,35

Square with side 30

The product of 30 by 1;24,51,10 is precisely 42;25,35.

Method of the mean. The method of the mean can easily be used
to find the square root of any number. The idea is simple: to find
the square root of 2, say, select x as a first approximation and take
for another 2/x. The product of the two numbers is of course 2, and
moreover, one must be less than and the other
√ greater than 2. Take the
arithmetic average to get a value closer to 2. Precisely, we have

1. Take a = a1 as an initial approximation.


√ √
2. Idea: If a1 < 2 then a21 > 2.
Babylonian Mathematics 14

3. So take
2
a2 = (a1 + )/2.
a1
4. Repeat the process.

Example. Take a1 = 1. Then we have


2 3
a2 = (1 + )/2 =
1 2
3 2 17
a3 = ( + )/2 = 1.41666... =
2 3/2 12
17 2 577
a4 = ( + )/2 =
12 17/12 408
Now carry out this process in sexagesimal, beginning with a1 = 1; 25
using Babylonian arithmetic without rounding, to get the value 1;24,51,10.

Note: 2=1;ú 25 = 1.4166... was commonly used as a brief, rough and
ready, approximation. When using sexagesimal numbering, a lot of
information can be compressed into one place.
Solving Quadratics. The Babylonian method for solving quadratics
is essentially based on completing the square. The method(s) are not
as “clean” as the modern quadratic formula, because the Babylonians
allowed only positive solutions. Thus equations always were set in a
form for which there was a positive solution. Negative solutions (indeed
negative numbers) would not be allowed until the 16th century CE.
The rhetorical method of writing a problem does not require vari-
ables. As such problems have a rather intuitive feel. Anyone could un-
derstand the problem, but without the proper tools, the solution would
be impossibly difficult. No doubt this rendered a sense of the mystic
to the mathematician. Consider this example

I added twice the side to the square; the result is 2,51,60.


What is the side?

In modern terms we have the simple quadratic x2 + 2x = 10300.


The student would then follow his “template” for quadratics. This tem-
plate was the solution of a specific problem of the correct mathematical
Babylonian Mathematics 15

type, all written rhetorically. Here is a typical example given in terms


of modern variables. Problem. Solve x(x + p) = q.
Solution. Set y = x + p
Then we have the system
xy = q
y−x = p
This gives
4xy + (y − x)2 = p2 + 4q
(y + x)2 = p2 + 4q
q
x+y = p2 + 4q
q
2x + p = p2 + 4q

−p + p2 + 4q
x =
2

All three forms


x2 + px = q
x2 = px + q
x2 + q = px

are solved similarly. The third is solved by equating it to the non-


linear system, x + y = p, xy = q. The student’s task would be to take
the problem at hand and determine which of the forms was appropriate
and then to solve it by a prescribed method. What we do not know is
if the student was ever instructed in principles of solution, in this case
completing the square. Or was mathematical training essentially static,
with solution methods available for each and every problem that the
practitioner would encounter.
It is striking that these methods date back 4,000 years!
Solving Cubics. The Babylonians even managed to solve cubic equa-
tions, though again only those having positive solutions. However, the
form of the equation was restricted tightly. For example, solving x3 = a
Babylonian Mathematics 16

was accomplished using tables and interpolation. Mixed cubics


x3 + x2 = a
were also solved using tables and interpolation. The general cubic
ax3 + bx2 + cx = d
can be reduced to the normal form
y 3 + ey 2 = g
To do this one needs to solve a quadratic, which the Babylonians could
do. But did the Babylonians know this reduction?
The Babylonians must have had extraordinary manipulative skills
and as well a maturity and flexibility of algebraic skills.
Solving linear systems. The solution of linear systems were solved
in a particularly clever way, reducing a problem of two variables to
one variable in a sort of elimination process, vaguely reminiscent of
Gaussian elimination. Solve
2 1
x − y = 500
3 2
x + y = 1800

Solution. Select x̃ = ỹ such that


x̃ + ỹ = 2x̃ = 1800
So, x̃ = 900. Now make the model
x = x̃ + d y = ỹ − d
We get
2 1
(900 + d) − (900 − d) = 500
3 2
2 1
( + )d + 1800/3 − 900/2 = 500
3 2
7
d = 500 − 150
6
6(350)
d =
7
So, d = 300 and thus
x = 1200 y = 600.
Babylonian Mathematics 17

Plimpton 322 tablet


Yale Babylonian collection

5 Pythagorean Triples.

As we have seen there is solid evidence that the ancient Chinese were
aware of the Pythagorean theorem, even though they may not have had
anything near to a proof. The Babylonians, too, had such an awareness.
Indeed, the evidence here is very much stronger, for an entire tablet of
Pythagoreantriples has been discovered. The events surrounding them
reads much like a modern detective story, with the sleuth being archae-
ologist Otto Neugebauer. We begin in about 1945 with the Plimpton
322 tablet, which is now the Babylonian collection at Yale University,
and dates from about 1700 BCE. It appears to have the left section
Babylonian Mathematics 18

broken away. Indeed, the presence of glue on the broken edge indi-
cates that it was broken after excavation. What the tablet contains is
fifteen rows of numbers, numbered from 1 to 15. Below we list a few
of them in decimal form. The first column is descending numerically.
The deciphering of what they mean is due mainly to Otto Neugebauer
in about 1945.
1.9834... 119 169 1
1.94915 3367 4825 2
..
.
1.38716 56 106 15

Interpreting Plimpton 322. To see what it means, we need a model


right triangle. Write the Pythagorean triples, the edge b in the col-
umn thought to be severed from the tablet. Note that they are listed

c
b

B
a

decreasing cosecant. Right Triangle

b (c/b)2 a c
120 (169/120)2 119 169 1
3456 (4825/3456)2 3367 4825 2
..
.
90 (106/90)2 56 106 15
c
csc2 B = ( )2
b
A curious fact is that the tablet contains a few errors, no doubt tran-
scription errors made so many centuries ago. How did the Babylonian
mathematicians determine these triples? Why were they listed in this
order? Assuming they knew the Pythagorean relation a2 + b2 = c2 ,
divide by b to get
a c
( )2 + 1 = ( )2
b b
Babylonian Mathematics 19

u2 + 1 = v 2
(u − v)(u + v) = 1
Choose u + v and find u − v in the table of reciprocals.

Example. Take u + v=2;15. Then u − v = 0; 26, 60 Solve for u and v


to get
u = 0; 54, 10 v = 1; 20, 50.
Multiply by an appropriate integer to clear the fraction. We get a = 65,
c = 97. So b = 72. This is line 5 of the table.

It is tempting to think that there must have been known general


principles, nothing short of a theory, but all that has been discovered
are tablets of specific numbers and worked problems.

6 Babylonian Geometry

Circular Measurement. We find that the Babylonians used π = 3


for practical computation. But, in 1936 at Susa (captured by Alexander
the Great in 331 BCE), a number of tablets with significant geometric
results were unearthed. One tablet compares the areas and the squares
Babylonian Mathematics 20

of the sides of the regular polygons of three to seven sides. For example,
there is the approximation
perimeter hexagon
= 0; 57, 36
circumference circumscribed circle
This gives an effective π ≈ 3 18 . (Not bad.)
Volumes. There are two forms for the volume of a frustum given

Frustum
b
b

a
a+b 2
V = ( )h
Ã2 !
a+b 2 1 a−b 2
V = h ( ) − ( )
2 3 2
The second is correct, the first is not.

There are many geometric problems in the cuneiform texts. For


example, the Babylonians were aware that

• The altitude of an isosceles triangle bisects the base.


• An angle inscribed in a semicircle is a right angle. (Thales)

7 Summary of Babylonian Mathematics

That Babylonian mathematics may seem to be further advanced than


that of Egypt may be due to the evidence available. So, even though
Babylonian Mathematics 21

we see the development as being more general and somewhat broader in


scope, there remain many similarities. For example, problems contain
only specific cases. There seem to be no general formulations. The lack
of notation is clearly detrimental in the handling of algebraic problems.

There is an absence of clear cut distinctions between exact and


approximate results.

Geometric considerations play a very secondary role in Babylonian alge-


bra, even though geometric terminology may be used. Areas and lengths
are freely added, something that would not be possible in Greek mathe-
matics. Overall, the role of geometry is diminished in comparison with
algebraic and numerical methods. Questions about solvability or in-
solvability are absent. The concept of “proof” is unclear and uncertain.
Overall, there is no sense of abstraction. In sum, Babylonian mathe-
matics, like that of the Egyptians, is mostly utilitarian — but apparently
more advanced.

8 Exercises

1. Express the numbers 76, 234, 1265, and 87,432 in sexagesimal.


2. Compute the products
(a) 1, 23 × 2, 9
(b) 2, 4, 23 × 3, 34
3. A problem on one Babylonian tablets give the base and top of an
isosceles trapezoid to be 50 and 40 respectively and the side length
to be 30. Find the altitude and area. Can this be done without the
Pythagorean theorem?
4. Solve the following system ála the Babylonian “false position”
method. State clearly what steps you are taking.
2x + 3y = 1600
5x + 4y = 2600
(The solution is (200, 400).)
Babylonian Mathematics 22

5. Generalize this Babylonian algorithm for solving linear systems to


arbitrary linear systems in two variables?
6. Generalize this Babylonian algorithm for solving linear systems to
arbitrary linear systems?

7. Modify the Babylonian root finding method (for 2) to find√the
square root of any number. Use your method to approximate 3.
Begin with x0 = 1.

8. Explain how to adapt the method of the mean to determine 3 2.
n n3 + n2
1 2
2 12
9. Consider the table: 3 36 Solve the following prob-
4 80
5 150
6 252
lems using this table and linear interpolation. Compare with the
exact values. (You can obtain the exact solutions, for example, by
using Maple: evalf(solve(x3 + x2 = a, x)); Here a=the right side)
(a) x3 + x2 = 55
(b) x3 + x2 = 257
10. Show that the general cubic ax3 + bx2 + cx = d can be reduced
to the normal form y 3 + ey 2 = g.
11. Show how the perimeter identity is used to derive the approxima-
tion for π.
12. Write a lesson plan wherein you show students how to factor
quadratics ála the Babylonian methods. You may use variables,
but not general formulas.
Ancient Greek Mathematics
The Greek Empire
• Ancient Greek civilization generally accepted to date from around 800 BC. Primarily centered
on the Aegean Sea (between modern-day Greece and Turkey) containing hundreds of islands
and loosely affiliated city-states.

• Many wars between city-states other empires (e.g. Persians).

• By 500 BC covered much of modern Greece, the Aegean and southern Italy. As a trading/sea-
faring culture, built/captured city-states (colonies/trading-outposts)all around the north and
east coast of the Mediterranean from Spain round the Black Sea and Anatolia (modern Turkey)
to Egypt.

• Alexander the Great (356–323 BC) extended empire around the eastern Mediterranean inland
capturing mainland Egypt and then east to western India and Babylon where he died.

• Eventually becomes part of the Roman Empire c.146 BC though Romans left Greek largely
essentially intact apart from crushing several rebellions.

• Greek civilization flourished even as the Rome collapsed, continuing as part of the Byzantine
Empire.

Map of Greek Empire c.500 BC from timemaps.com

Ancient Greece is important for far more than just mathematics and one course cannot begin to
do justice to it. Much of modern western thought and culture including philosophy, art logic and
science has roots in Ancient Greece. While undeniably important, western culture has often over-
emphasized the role of the Greeks and downplayed the contribution of other cultures to our inherited
knowledge.
Mathematics and Philosophical Development
• Inquiry into natural phenomena encouraged through the personification of nature (sky = man,
earth = woman) which pervaded early religion.
• By 600 BC ‘philosophers’ were attempting to describe such phenomena in terms of natural
causes rather than being at the whim of the gods. For example, all matter was suggested to be
comprised of the four elements (fire, earth, water, air).
• Development of Mathematics linked to religion (mysticism/patterns/assumption of perfec-
tion in the gods’ design), philosophy (logic) and natural philosophy (description of the natural
world). Mathematics elevated from purely practical considerations to an extension of logic: the
Greeks were unhappy with approximations even when such would be perfectly suitable for
practical use. Led to the development of axiomatics and proof.
• Limited extant mathematics from pre-300 BC. Most famous work is Euclid’s Elements c.300 BC,
indisputably the most important mathematical work in western mathematics and a primary
textbook in western education until the early 1900’s. Probably a compilation/editing of earlier
works, its importance meant other works were sacrificed and sometimes subsumed by it.
• The word theorem (theory, theorize, etc.) comes from the Greek theoreo meaning I contemplate. A
theorem is therefore an observation based on contemplation.
• Later mathematics included Ptolemy’s Almagest c.150 AD on Astronomy and the forerunner of
trigonometry: basis of western astronomical theory until the 1600’s.

Enumeration
The ancient Greeks had two primary forms of enumeration, both developed c.800–500 BC.

Attic Greek (Attica = Athens): Strokes were used for 1–4. The first letter of the words for 5, 10, 100,
1000 and 10000 denoted the numerals. For example,
• πeντe (pente) is the Greek word for five, whence Π denoted 5.
• δeκα (deca) means ten, so ∆ = 10.
• H (hekaton), X (khilias) and M (myrion/myriad) denoted 100, 1000 and 10000 respectively.
• Combinations were used, e.g. ∆∆ΠΠ||| = 223.
The construction of large numbers was very similar to the more familiar Roman numeral system.
Ionic Greek (Ionia = middle of Anatolian coast): the alphabet
denoted numbers 1–9, 10–90 and 100–900 in the same way 1 α 10 ι 100 ρ
as Egyptian hieratic numerals were formed. The alphabet 2 β 20 κ 200 σ
differs from modern Greek due to three archaic symbols ϛ, 3 γ 30 λ 300 τ
ϙ, ϡ (stigma, qoppa, sampi). 4 δ 40 µ 400 υ
Larger numbers used a left subscript to denote thousands 5 ε 50 ν 500 φ
and/or M (with superscripts) for 10000, as in Attic Greek. 6 ϛ 60 ξ 600 χ
For example, 7 ζ 70 o 700 ψ
8 η 80 π 800 ω
γ
35298 =,λ,εσϙη = M,εσϙη 9 θ 90 ϙ 900 ϡ

2
Eventually a bar was placed over numbers to distinguish them from words (e.g. ξθ = 89). Modern
practice is to place an extra superscript (keraia) at the end of a number: thus 35298 = ͵λ͵εσϙηʹ
Reciprocals/fractions were denoted with accents: e.g. θ́ = 19 . The use of Egyptian fractions persisted
in Europe into the middle ages.

Both systems were fine for record-keeping but terrible for calculations! Later Greek mathematicians,
in particular Ptolemy, adapted the Babylonian sexagesimal system for calculation purposes thus ce-
menting the use of degrees in astronomy and navigation.

Early (pre-Euclidean) Greek Mathematics

Euclid’s Elements forms a natural breakpoint in Greek mathematical history; almost everything that
came before the Elements was eventually swallowed by it. Pre-Euclidean mathematics is therefore
largely a discussion of the origins of some of the ideas in Euclid.

Thales of Miletus (c.624–546 BC)

Often thought of as the first western scientist, Thales is also important in mathematics.

• Olive Trader based in Miletus, a city-state in Anatolia.


• Contact with Babylonian traders/scholars probably led to his learning some geometry and an
attempt to organize his discoveries.
• Stated some of the first abstract propositions: in particular,

– The angles at the base of an isosceles triangle are equal.


– Any circle is bisected by its diameter.
– A triangle inscribed in a semi-circle is right-angled (still known as Thales’ Theorem).

• Proofs not forthcoming. The major development was the stating


of abstract general principles. Thales’ propositions concern all
triangles, circles, etc. The Babylonians and Egyptians were merely
observed to use certain results in calculations and gave no indica-
tion that they appreciated the general nature of their results.

• Mathematical reasoning, if he conducted such, was almost cer-


tainly visual. For example, by 425 BC, Socrates could describe how
to halve/double the area of a square by joining the midpoints of
edges.
• Thales arguably more important to the history of reasoning: offered arguments/ discussions
concerning the ‘stuff’ of which the universe is made.

3
Pythagoras of Samos c.572–497 BC
• Much travelled (Egypt, Asia, Babylon, Italy) though his story was probably over-emphasised
after his death. Eventually settled in Croton (southeast Italy) where he founded a school/cult,
persisting over 100 years after his death. Mathematical results/developments came from the
group collectively.
• More of a mystic/philosopher than a mathematician. Core belief that number is fundamental
to nature. Motto: “All is number”. Emphasised form, pattern, proportion.
• Pythagoreans essentially practiced a mini-religion (they were vegetarians, belived in the trans-
migration of souls, etc.).
• The following quote1 helps give a flavor of the Pythagorean way of life.

After a testing period and after rigorous selection, the initiates of this order were al-
lowed to hear the voice of the Master [Pythagoras] behind a curtain; but only after
some years, when their souls had been further purified by music and by living in
purity in accordance with the regulations, were they allowed to see him. This pu-
rification and the initiation into the mysteries of harmony and of numbers would
enable the soul to approach [become] the Divine and thus escape the circular chain
of re-births.

Several famous results are attributable to the Pythagoreans. They were particularly interested in
musical harmony and the relationship of such to number. For instance, they related intervals in
music to the ratios of lengths of vibrating strings:

• Identical strings whose lengths are in the ratio 2:1 vibrate an octave apart.

• A perfect fifth corresponds to the ratio 3:2.

• A perfect fourth corresponds to the ratio 4:3.

Using such intervals to tune musical instruments (in particular pianos) is still known as Pythagorean
tuning.

Theorems 21–34 in Book IX of Euclid’s Elements are Pythagorean in origin:

Theorem (IX.21). A sum of even numbers is even.

Theorem (IX.27). Odd less odd is even.

The Pythagoreans studied perfect numbers: equal to the sum of their proper divisors (e.g. 6 =
1 + 2 + 3). They seem to have observed the following, though it is not known if they had a proof.

Theorem (IX.36). If 2n − 1 is prime then 2n−1 (2n − 1) is perfect.

They also considered square and triangular numbers and tried to express geometric shapes as num-
bers, all in service of their belief that all matter could be formed from the combination of basic shapes.
Cultish the Pythagoreans may have been, but they discovered many things and certainly had lofty
goals!
1 Van der Waerden, Science Awakening pp 92–93

4
Length, Number, Incommensurability and Pythagoras’ Theorem

Our modern notion of continuity facilitates tight relationship between length and number: any object
can be measured √ with respect to any fixed length. For instance, we’re happy stating that the diagonal
of a square is 2 times the side. To the Greeks and other ancient cultures, the only numbers were
positive integers. Appreciating the distinction between length and number is crucial to understanding
several of the major ideas of Greek mathematics. In particular, it helps explain the primacy of Ge-
ometry in their mathematics: lengths are real things that the Greeks wanted to compare using numbers.

A core Pythagorean belief was that of commensurability: given two lengths, there exists a sub-length
dividing exactly into both. One could then describe the relationship between lengths with a ratio.
For instance, if the longer length contained three of the sub-lengths and the smaller two, this could
be expressed with the ratio 3 : 2. In modern language,

∀ x, y ∈ R+ , ∃k, l ∈ N, r ∈ R+ such that x = kr and y = lr

This is complete nonsense for it insists that every ratio of real numbers is rational!
The supposition of commensurability clearly fits with the Pythagoreans’ mystical emphasis on the
perfection of number. The discovery that it was false produced something of a crisis. A (possibly)
apocryphal story suggests that a disciple named Hippasus (c.500 BC) was set adrift at sea as punish-
ment for revealing it. Nevertheless, it is generally accepted that the Pythagoreans provided the first
evidence of the existence of irrational numbers in the form of incommensurable lengths.

By 340 BC, Aristotle was happy to state that incommensurable lengths exist.

Theorem (Aristotle). If the diagonal and side of a square are commensurable, then odd numbers equal even
numbers.

Inferred proof—original unknown. Consider Socrates’ doubled-


square from earlier. Label the sides of the blue square a and the
a a
diagonal b where a and b are integers denoting multiples of the
common sub-length (the existence of a common sub-length is the
hypothesis!).
b
We may assume that at least one of a or b is odd, for otherwise
there exists a larger common sub-length. However the larger
square (side b) is twice the smaller (side a) so b is even and the
square number b2 is divisible by four. But then a is also even.
Whichever of a, b was odd is also even: contradiction!

Note the similarity of this argument2 to the standard modern proof of the irrationality of 2

2 For those with musical training, a similar argument shows that the Pythagorean notion of perfect fifths in music is also

flawed. The cycle of fifths is a musical principal stating that an ascension through twelve perfect fifths takes you through
every note in the standard chromatic scale, finishing seven octaves above where you start. This is essentially a claim that
 12
27 = 23 ⇐⇒ 219 = 312 : palpable nonsense!

5
While there is no evidence that the Pythagoreans ever provided a correct proof of their famous The-
orem, one argument possibly attributable to the Pythagoreans used the idea of commensurability.
‘Proof’ of Pythagoras’ Theorem. Label the right triangle a, b, c
where c is the hypotenuse and drop the altitude to the hy-
potenuse. Let d be the length of the a-side of the hypotenuse.
Similar triangles tell us that

a : d = c : a =⇒ a2 : ad = cd : ad =⇒ a2 = cd (∗) a b

Thus the square on a has the same area as the rectangle below
d c
the d-side of the hypotenuse. Repeat the calculation on the other
side to obtain b2 = c(c − d). Now sum these for the proof.

In the language of the Pythagoreans, the only acceptable numbers


were integers, so the symbols a, b, c, d in were the integer multiples
of an assumed common sub-length. This restriction completely
destroys the generality of the proof.
It is clear from the organization of Book I of Euclid’s Elements that one of its primary goals was
to provide a rigorous proof of Pythagoras’ Theorem which did not depend on the flawed notion of
commensurability. With our modern understanding of real numbers, there is nothing wrong with the
above argument. We have fully adapted to the idea that length and number are freely interchangeable
due to the completeness of the real numbers: ancient mathematicians had no such knowledge.

Zeno of Elea c.450 BC


Zeno might be called the patron saint of devil’s advocates. His fame comes from his suggestion
of several ingenious arguments/paradoxes involving the infinite and the infinitesimal. Here are
perhaps the two most famous of these.
• Achilles and Tortoise: Achilles starts a race behind a Tortoise. After a given time t1 , Achilles
reaches the Tortoise’s starting position, but the Tortoise has moved on. After another time
interval t2 , Achilles reaches the Tortoise’s second position: again the Tortoise has departed. In
this manner Achilles spends an infinite sum of time intervals t1 + t2 + t3 + · · · in the chase.
Zeno’s paradoxical conclusion: Achilles never catches the Tortoise.
The problem was that Zeno refused to accept that the total duration could be finite, even though
it be split into infinitely many subintervals of time. The resolution of this paradox is at the heart
of the modern notion of infinite series.
• Arrow paradox: An arrow is shot from a bow. At any given instant the arrow doesn’t move. If
time is made up of instants, then the arrow never moves.
This time Zeno is debating the the idea that a finite time period can be considered as a sum of
infinitessimal instants. Again the problem is one of limits and infinite sums.
Zeno’s paradoxes have stimulated philosophers for thousands of years, continuing well into the 18th
century as the theory of calculus was fleshed out. We shall revisit this controversy later. For the
present, it is enough to consider how radical the fundamental ideas of calculus are: to measure an
area one essentially slices it into infinitely many, infinitesimally thin strips before summing them up.
Try selling this idea to someone who has never studied calculus!

6
Governors State University
OPUS Open Portal to University Scholarship
All Student Theses Student Theses

Spring 2016

History of Mathematics from the Islamic World


Asamah Abdallah
Governors State University

Follow this and additional works at: http://opus.govst.edu/theses


Part of the Islamic World and Near East History Commons, and the Mathematics Commons

Recommended Citation
Abdallah, Asamah, "History of Mathematics from the Islamic World" (2016). All Student Theses. Paper 71.

For more information about the academic degree, extended learning, and certificate programs of Governors State University, go to
http://www.govst.edu/Academics/Degree_Programs_and_Certifications/

Visit the Governors State Mathematics Department


This Thesis is brought to you for free and open access by the Student Theses at OPUS Open Portal to University Scholarship. It has been accepted for
inclusion in All Student Theses by an authorized administrator of OPUS Open Portal to University Scholarship. For more information, please contact
opus@govst.edu.
History of Mathematics from the Islamic World

By

Asmah Abdallah
B.S., Governors State University, 2012

Thesis
Masters Project

Submitted in partial fulfillment of the requirements

For the Degree of Masters of Science


With a Major in Mathematics

Governors State University


University Park, IL 60484

Fall 2015
TABLE OF CONTENTS

Abstract ........................................................................................................................................................ 1

Introduction .................................................................................................................................................. 2

Al-Khwarizmi on Algebra............................................................................................................................... 3

Basic Ideas in Al-Khwarizmi’s Algebra.............................................................................................. 4

Abu-Kamil on Algebra .................................................................................................................................. 8

Illustration on Roots… .................................................................................................................... 10

Rule of False Position ..................................................................................................................... 11

Al-Uqlidisi on Hindu Arithmetic ................................................................................................................ 13

Kushyar ibn Labban’s Principle of Hindu Reckoning ................................................................................... 16

Al-Khayyam ................................................................................................................................................. 21

Al-Khayyam on the Reform of the Persian Calendar … ................................................................. 22

Applications .............................................................................................................................................. 23

Inheritance .................................................................................................................................. 24

Conclusion ................................................................................................................................................ 25

References ................................................................................................................................................. 27
Abdallah 1

Abstract

"The early history of the mind of men with regard to mathematics leads us to point out

our own errors; and in this respect it is well to pay attention to the history of mathematics." A De

Morgan [17]. Learning the history of mathematics is crucial to fully understanding the world of

mathematics today. This paper will explore the history of mathematics from the Islamic world. It

will focus on the contributions of well-recognized mathematicians including, Al-Khwarizmi, Al-

Khayyam, Uqlidisi, Kushyar ibn Labban, and Abu Kamil. It will also concentrate on the

contributions that the Islamic world had on algebra, beginning with Al-Khwarizmi and his

contribution to the developmental of algebraic equations, and Khayyam and his contribution to

the geometrization of algebra. This paper will also discuss the ways in which the Muslims

applied the mathematics they learned into their lives. This paper will provide its readers with a

strong foundation on the history of math from the Islamic world which will better enable its

readers to fully understand the mathematics we use today.


Abdallah 2

Prophet Muhammad (peace be upon him) stated, “Seeking knowledge is a duty on every

Muslim” (Bukhari) [23]. Thus, there are many Muslim scholars who were keen on doing their

part. From the 9th-15th century, Islamic science and mathematics flourished. Throughout history,

Muslims from different parts of the world have contributed to the development of mathematics.

One way this was done was by translating all sorts of knowledge they believed would be

beneficial to society. The two main sources the Muslims translated were the works of the Hindus

and the Greeks. Thabit ibn Qurra, a Muslim mathematician, translated the works written by

Euclid, Archimedes, Apollonius, Ptolemy, and Eutocius. In Baghdad during 810 A.D, he also

founded The House of Wisdom, a school which was dedicated to translating books from Greek

to Arabic and also creating commentaries on these books. Thanks to these translations, the

knowledge of the ancient Greek texts has survived to this day.

Muslim mathematicians have made significant contributions to different parts of

mathematics including algebra, geometry, trigonometry, calculus, arithmetic, and so on. The

number system and decimal point we use today comes from the Islamic world. Connected to the

decimal system come the fundamental operations: addition, subtraction, multiplication, and

division, exponentiation, and extracting the root; although these fundamental operations are

possible without the use of the Hindu-Arabic decimal system. They are also responsible for the

invention of sine and cosine, the ruler, and the compass. The word algebra comes from “Al-

Jabr”, which comes from the book written by Muhammad ibn Musa Khwarizmi, Hisab al-Jabr

wa Muqabala. Al-Khawarizimi was the first to introduce the concept of zero, also known as

“cipher” in the Arabic language. De Vaux, a prominent historian stated the following, “By using

ciphers, (Arabic for zero) the Arabs became the founders of the arithmetic of everyday life; they

made algebra an exact science. The Arabs kept alive higher intellectual life and the study of
Abdallah 3

science…” [23] The chart below shows the numbers we use today. Below are the Hindu-Arabic

numbers, compared to the number written in the Arabic language.

We will now look at some prominent mathematicians that have contributed greatly to the

development of mathematics.

Al-Khwarizmi on Algebra
Abdallah 4

Muhammad ibn Musa al-Khwarizmi was born around 780 AD in Baghdad and died

around 850 AD. He was a Muslim mathematician and astronomer, who was known for his major

contribution on Hindu-Arabic numerals and concepts in algebra, which we will discuss in more

detail. Al-Khwarizmi was one of the first to use zero as a place holder in positional base

notation. The word algorithm actually derives from his name.

Al-Khwarizmi was most known for his book on elementary algebra, Al-Kitāb Al-

Mukhtaṣar fī Hisāb Al-Jabr Waʾl-muqābala (“The Compendious Book on Calculation by

Completion and Balancing”) which is considered one of the first books to be written on algebra.

He also wrote a book where he introduces the Hindu-Arabic numerals and their arithmetic. His

third major book, Kitāb ṣūrat al-arḍ (“The Image of the Earth”) presents the coordinates of

localities in the known world, including locations in Africa and Asia. Al-Khwarizmi assisted in

the construction of a world map, participated in the investigation of determining the

circumference of the Earth, and he found volumes of figures such as spheres, cones, and

pyramids. He also compiled a set of astronomical tables based on Hindu and Greek sources.

Most of Al-Khwarizmi’s work was translated into Latin.

Basic Ideas in Al-Khwarizmi’s Algebra

According to Al-Khwarizmi, there are three types of quantities: simple numbers (which

we would refer to today as natural numbers), such as 1, 18, and 105; root numbers, which he

considers an unknown values and calls them “things” (which we would denote today as 𝑥); and

wealth, which is the square of the root or unknown, also known as mal. This is usually denoted

as 𝑥 2 . Also, he states the six basic types of equations as:


Abdallah 5

1) Roots equal numbers(𝑛𝑥 = 𝑚).

2) Wealth equal roots (𝑥 2 = 𝑛𝑥).

3) Wealth equal numbers (𝑥 2 = 𝑚).

4) Numbers and wealth equal roots (𝑚 + 𝑥 2 = 𝑛𝑥).

5) Numbers equal roots and wealth (𝑚 = 𝑛𝑥 + 𝑥 2 ).

6) Wealth equals numbers and roots (𝑥 2 = 𝑚 + 𝑛𝑥).

We will now look at an example from Al-Khwarizmi’s work.

Example 1: Solve 𝑥 2 + 21 = 10𝑥

Note: Nowadays, we would simply solve this quadratic equation by using what we call the
−𝑏±√𝑏 2 −4𝑎𝑐
quadratic formula, , or by factoring if the problem is factorable. We can also graph the
2𝑎
function and use graphing as a method to solve.

Solution: The first procedure Al-Khwarizmi uses in solving this problem is show in Fig. 1,

where he first halves the number of roots, where he receives 5. He then multiplies 5 by itself,

where he receives 25. Next, he subtracts 21 from this product, where he receives 4. Further, he

takes the square root of 4, where he obtains 2, and subtracts that from 5, where he then receives

3.

10 10 2
− √( 2 ) − 21 [Fig. 1]
2

In his second procedure, he takes the exact same steps as in procedure 1, however, this time

instead of taking half the roots and subtracting, he takes half the roots and adds this time. This

yields the following expression, as shown in figure 2.

5 + √52 − 21 [Fig. 2]
Abdallah 6

The solution to figure 2 yields 7. In this procedure, he refers to the 10 as “the number of roots”,

and 21 as the simple number.

Al-Khwarizmi describes the general solution of any quadratic equation of type 4 (as

shown above), where n represents the number of roots and m represents any number as the

following…

𝑛 𝑛 2
± √(2 ) − 𝑚 [Fig. 3]
2

He stated that there were no solutions whenever he received a number less than zero under the

square root. Nowadays, we call these numbers imaginary. He also acknowledges that when the

number under the square root is equal to zero, then only one solution exists. Also, whenever Al-

𝑚 𝑛
Khwarizmi had a coefficient in front of 𝑝𝑥 2 , he would divide by p, obtaining 𝑥 2 + ( 𝑝 ) = (𝑝) 𝑥.

This shows that his coefficients were not restricted to whole numbers only.

We will now turn to another example focusing on the fifth basic types of equation. In this

example, we have 39 = 𝑥 2 + 10𝑥, where we have the number equals roots and wealth. Al-

Khwarizmi uses an algebraic proof and a geometric proof. We will first look at the algebraic

proof which is as follows: The first step is to take half of the roots, 10, which gives us 5. We then

multiply it by itself, which is 25. We then add this to 39, where we receive 64. We take the square

root of 64, which is 8 and subtract it from it half the roots, 5, which leaves us with 3, our solution.
Abdallah 7

[Fig. 4]

Next, we will take a look at his geometric proof. In the first step, Al-Khwarizmi starts with a

square, where each side length is represented by x. Therefore, the area of the square is 𝑥 2

(figure 4). Now that we have 𝑥 2 , we must now add 10𝑥. We do this by adding four rectangles,

10 5
each or in length and length x to the square. Here we now have 𝑥 2 + 10𝑥, which in our
4 2

example equals to 39 (figure 4). Last, Al-Khwarizmi finds the area of the four little squares,

5 5 25 25
which is 2 × 2 which gives us . Thus, the outside square of figure 4 has an area of × 4 + 39
4 4

25
since the area of the 4 squares are and we have the 𝑥 2 + 10𝑥 left which we already know is
4

equal to 39. Solving for the area, we receive 25 + 39, which equals 64. Therefore, the side

5 5
length of the square is 8, since the square root of 64 is 8 . The side length is equal to 2 + 𝑥 + 2.
Abdallah 8

5
This can be seen from figure 4 where the two squares have a side length of 2. Therefore,

𝑥 + 5 = 8, so 𝑥 = 3. This technique works because once we find the area of the square above,

we can use that to determine what the x-value would equal by determining its square root.

Abu Kamil on Algebra

Abu Kamil Shuja ibn Aslam was born in about 850 AD, most likely in Egypt, and died in

930 AD. He was a Muslim mathematician who was referred to as the “Egyptian Calculator”

during the Islamic Golden Age, which was a period that occurred during the middle ages in

which much of the historical Arab world experienced a scientific and economic flourishing. It

occurred during the 8th century until about the mid 13th century. Abu Kamil is considered to be

the first mathematician to use and accept irrational numbers as solutions and as coefficients to

equations. Leonardo Bonacci, a twelfth century European mathematician, adopted his

mathematical techniques, which allowed Abu Kamil to play an important role in introducing

algebra to Europe, even after his death. He worked on and solved non-linear simultaneous

equations with three unknown variables. Abu Kamil was one of the first Muslim

Mathematicians to work with powers higher than two; the highest power he worked with was the
Abdallah 9

eighth power. He understood that 𝑥 5 can be expressed in terms of squares, as 𝑥 2 𝑥 2 𝑥. For 𝑥 6 , he

used cubes and expressed it as 𝑥 3 𝑥 3 .

Abu Kamil wrote many books on mathematics during his lifetime. Some of these books

include, but are not limited to the following: Kitāb fī al-jabr wa al-muqābala (Book of Algebra),

Kitāb al-ṭarā’if fi’l-ḥisāb (Book of Rare Things in the Art of Calculation), Kitāb al-mukhammas

wa’al-mu‘ashshar (On the Pentagon and Decagon), and Kitāb al-misāḥa wa al-handasa (On

Measurement and Geometry). In his first book, Book of Algebra, Abu Kamil discusses and

solves problems including, but not limited to, the application of geometry dealing with unknown

variables and square roots, quadratic irrationalities, polygons, indeterminate equations, and

recreational mathematics. His book, Book of Rare Things in the Art of Calculation, provides a

number of procedures on finding integral solutions and indeterminate equations. In On the

Pentagon and Decagon, Abu Kamil calculates the numerical approximation for the side of a

regular pentagon in a circle. Lastly, his book On Measurement and Geometry contains a set of

rules for calculating the volume and surface area of solids. We will now look at some of the

examples in his work.

Abu Kamil demonstrates rules and properties of numbers such as 𝒂𝒙 × 𝒃𝒙 = 𝒂𝒃 × 𝒙𝟐

and 𝒂 × (𝒃𝒙) = (𝒂𝒃) × 𝒙. He also shows an example of the distributive property where he

shows that: (𝟏𝟎 − 𝒙) × (𝟏𝟎 − 𝒙) = 𝟏𝟎𝟎 + 𝒙𝟐 − 𝟐𝟎𝒙. Abu Kamil solves this problem

algebraically and geometrically, we will look at his geometric proof.

Proof: In figure 5, let line GA be equivalent to 10 in length and GB, 𝑥.


Abdallah 10

[Fig. 5]

By constructing the square AD on the segment GA, we will get that AB = ED = 𝟏𝟎 −

𝒙. Therefore, the square (ZH) = (𝟏𝟎 − 𝒙)𝟐 , and (GZ) = (GH) = 𝟏𝟎𝒙. Hence, (EH) = (GH) – (EB)

= 𝟏𝟎𝒙 − 𝒙𝟐 . Therefore, we have that (EH) + (GZ) = 𝟐𝟎𝒙 − 𝒙𝟐 and we know that the large

square is 100 so we have the following:

(𝟏𝟎 − 𝒙)𝟐 = (𝒁𝑯) = 𝟏𝟎𝟎 − (𝟐𝟎𝒙 − 𝒙𝟐 ) = 𝟏𝟎𝟎 + 𝒙𝟐 − 𝟐𝟎𝒙.

Abu Kamil’s Illustration on Roots

Assume we have the problem: a square is equal to five of its roots, 𝒙𝟐 = 𝟓𝒙. The root of

the square is always equal to the roots to which the square is equal to, in our case, 𝟓𝒙. For 𝒙𝟐 , we

draw a square, abgd, and then divide it into 5 equal rectangles, as shown if figure 6.

[Fig. 6]
Abdallah 11

Take note that lines be, ek, kr, rh, and hg are all equivalent and all equal to 1. Therefore, the line

bg is equivalent to 5. Hence, the area of the square is 25, and from the figure above we can see

that the square of 25 is 5. If we multiply the side length ab by the side length be, that would give

us the surface of abec, which is the root of the square abgd. The surface of the square abgd is

equivalent to five times the root of itself, or five roots.

Abu Kamil on the Rule of False Position

To solve the following problem, Abu Kamil uses the algebraic device known as “the rule

of false position”, which is the term used for the method used to evaluate a problem by using

“test”, or false values for the given variables, and then adjusting them accordingly. The problem

below will show us an example of this.

Example 1: Find a quantity that if increased by its seventh part is equal to 19.

1
Solution: We have the following algebraic equation: 𝑥 + 7 𝑥 = 19. Using false position, we plug

in 7 for x (we use x because it is easy to work with since it eliminates our fraction) and obtain the
1
following: 7 + 7 × 7 which equals 8, rather than 19. Therefore, we will then divide 19 into 8 and

19 𝑥
then multiply the result by 7. We will set this as the following proportion: = 7. The reason we
8

do this is because when we plug in 7 the receive 8 as a solution. So the question remains what

must we plug in, in order to receive 19. Once we set up this proportion and solve it for x, we

receive 16.625.

Let’s look at another example using false position.


Abdallah 12

Example 2: Solve the systems of equations: 7𝑦 = 13𝑥 + 4 (1)

4𝑦 = 2𝑥 + 176 (2)

Solution: We will use false position and have 𝑦1 = 40. Plugging in for equation (1) we

3
receive 7(40) = 13𝑥 + 4. Solving for x, we receive that 𝑥1 = 21 13. Then, we plug in 𝑥1 and 𝑦1

3
into the second equation where we receive 4(40) = 2(21 13) + 176, where we get that 160 =

6 6 6
218 13. Next, we find the difference of these two numbers: 218 13 − 160 = 58 13 = 𝑑1 .

Using false position again, we will plug in 80 for 𝑦2. Plugging in for equation (1) we

10
receive 7(80) = 13𝑥 + 4. Solving for x, we receive that 𝑥2 = 42 13. Then, we plug in 𝑥2 and 𝑦2

10
into the second equation where we receive 4(80) = 2(42 13) + 176, where we get that 320 =

7 7 6
261 13. Next, we find the difference of these two numbers: 261 13. − 320 = −58 13 = 𝑑2 . The

last step is to solve for y by doing the following: We take our 𝑦2 and multiply it by our 𝑑1 and

multiply our 𝑦1 and 𝑑2 . Then we find the difference between the two. Once we have that, we

divide this number by the difference of 𝑑1 and 𝑑2 . This can be seen by the following equation:

6 6
80 (58 13) + 40(58 13)
𝑦= 6 6 = 60
(58 + 58 )
13 13

Replacing y with 60 in equation (1), we receive that x is equal to 32.


Abdallah 13

Al-Uqlidisi’s on Hindu Arithmetic

Abu'l Hassan Ahmad ibn Ibrahim Al-Uqlidisi’s was an Arab mathematician who was

born around 920 AD in Damascus and died in 980 AD in Damascus. He traveled widely and met

and studied from many mathematicians he met throughout his traveling. He was the author of

Kitab al-Fusul fi al-Hisab al-Hindi (The Book of Chapters on Hindu Arithmetic) and Kitab al-

hajari fi al-hisab(The Book of Records on Arithmetic). In his work, Uqlidisi focuses on the

positional use of Arabic numerals and decimal fractions, where we will look at a couple of his

examples below. His treaty on arithmetic is divided into four sections.

In the first part of the treaties, Uqlidisi introduces the Hindu numerals and explains the

place value system. He describes addition, multiplication and other arithmetic operations on

integers and fractions in decimal notation. In the second part of the treatise he collects

arithmetical methods given by earlier mathematicians and converts them in the Indian system. In

the third part, Uqlidisi answers questions the reader may have such as “why do it this way?” or

“how can I solve this?” and so on. Some of these questions involve understanding the

justification in performing several arithmetic steps involved in manipulating problems. Other

examples of some of the questions asked are “how do we check what we need to check” or “how

do we extract roots of numbers”. In the last part, he claims that up to this work, the Indian

methods have been used with a blackboard in order to erase and move numbers around as the

calculation of the numbers took place. He also showed how to modify these methods when using

pen and paper.


Abdallah 14

Al-Uqlidisi’s work is one of the earliest known texts on how to deal with decimal

fractions. For example, to halve 19 successively, Al-Uqlidisi wrote the following: 9.5, 4.75,

2.375, 1.1875, and 0.59375. Another example of Uqlidisi is where he increases 135 by its tenth,

1
then the result by its tenth, etc. five times. He first starts by writing 135 × (1 + 10). Next,

135×11
changing the mixed number to an improper fraction, he receives . He then gets 148.5. Next
10

1 148.5 ×11 11 11
he gets 148.5 × (1 + 10), which equals to . He splits this up as 148 × 10 and 0.5 × 10.
10

11 11
He calculates 148 × 10, which equals 162.8 and 0.5 × 10 , which equals 0.55. He adds them to

get 163.35, which is his answer.

After studying Uqlidisi’s works, Saidan stated, “The most remarkable idea in this work is

that of decimal fraction. Al-Uqlidisi uses decimal fractions as such, appreciates the importance

of a decimal sign, and suggests a good one”. [3]

Al-Uqlidisi’s was recorded to discover the multiplication of two mixed numbers. He

changed the mixed numbers into improper fractions and multiplied across. In the example below,

we will show exactly how Uqlidisi multiplied two mixed numbers.

Example 1: Multiply 7 and a half by 5 and a third. What is shown below shows how Uqlidisi set

up such problems.

7 by 5

1 1
2 3
Abdallah 15

15
To solve, we first multiply 7 and 2 and add the one, which becomes . We then multiply
2

16
5 and 3 and add the one, which becomes 3 . Next, we multiply 15 and 16; receiving 240, then we

divide by 6, which gives us 40.

Here, Uqlidisi is simply changing a mixed number into an improper fraction, then

multiplying the numerators across and the denominators across. We use this exact method today;
1 1
we only set up the problem a bit differently. We would write this problem as 7 2 × 5 2.

1 1 1 1
Example 2: Multiply 19 + 3 + 4 by 13 + 2 + 5

13
19
1 1
1 1
2 5
3 4

1 1 1 1
To solve, we first add the fractions 3 with and with 5. To add the fractions, early
4 2

mathematicians would find would find a new denominator, which was done by finding the

product of the given denominators, which in our case is 3 and 4 and 2 and 5. After adding the
7 7
numerators, the fractions were then reduced to lowest terms. Here, we receive 12 and , which
10

we write as…

19 13
by
7 7
12 10
Abdallah 16

Now, we can simply solve this problem as we have solved the problem in example 1. The

outcome would be 32,195 out of 120.

Another way to solve this problem is to multiply the 19 by the product of 3 and 4, then

add the sum of 3 and 4 to that product and write it over the product of 3 and 4. We do the same

to the other; we multiply 13 by the product of 5 and 2, then add the sum of 5 and 2 to that

product and write it over the product of 5 and 2. The reason this works is because by multiplying

the whole number by the product of the denominators, we are simply multiplying by a common

denominator. Then the reason why we add the sum of the denominators is because if we multiply

the fractions by a common denominator, we end up getting both numbers in the numerator,

where we would add them (this only applies when we have a 1 in the numerator).

Kushyar ibn Labban’s Principles of Hindu Reckoning

Kushyar ibn Labban was a Persian mathematician, geographer, and astronomer born in

Gilan in 971 AD and thought to have died in Baghdad in 1029 AD. His main work seems to have

taken place during the 11th century. In one of Labban’s most major works,

the Jāmiʿ Zīj (Universal/Comprehensive astronomical handbook with tables), which was

influenced by Ptolemy's Almagest and al‐Battānī's Zīj, contains many tables concerning
Abdallah 17

trigonometry, astronomical functions, star catalogs, and geographical coordinates of cities. It

comprises four books: calculations, tables, cosmology, and proofs.

One of his most significant contributions was his work on Hindu reckoning. It is

described as follows: “Kushyar ibn Labban's Principles of Hindu reckoning ... is singularly

important in the history of mathematics, not only for its mathematical content, but also for its

linguistic interest and its relation to earlier and succeeding algorisms. It may be the oldest Arabic

mathematical text using Hindu numerals, and ibn Labban's concepts reveal considerable

originality...” [14] In the Principles of Hindu Reckoning, ibn Labban focuses on decimal

numbers and discusses the addition, subtraction, multiplication and division of numbers

involving decimals. He also provides different methods on constructing exact square roots, as

well as approximate methods to calculate the square roots of non-square numbers. He also does

the same for exact cube roots and cube root of a non-square number.

In Principles of Hindu Reckoning, Labban focuses on different arithmetic operations of

numbers and fraction. We will look at a few of his examples. It is important to take note that

many of these problems were done on dust boards, making it easy to erase and replace numbers

as shown in the examples below.

Example 1: Add 839 to 5625

We write it as follows…

5625
839 [Fig. 7]

We make sure that all our place values are lined up accordingly.
Abdallah 18

The first step is to add the highest place value common to both numbers. In this example it

would be 56 and the 8, where we receive 64. We replace the 56 with the 64 as shown in figure 8.

6425
839 [Fig. 8]

Next, we add the 3 and the 2, where we receive 5. We replace the 2 with the 5 as shown in figure

9.

6455
839 [Fig. 9]

Last, we add the 9 to the 5, where we receive 14. We add the 1 to the 5 in the tens place of 6455

and replace the 5 in the ones place with the 4 where we receive our final solution. This is shown

in figure 10.

6464
839 [Fig. 10]

Example 2: Subtract 839 from 5,625.

We write it as follows…

5625
839 [Fig.11]

The first step is to subtract 8 from 6; however, because this is not possible, instead, we subtract 8

from 56, where we receive 48. Hence, this yields the following figure…

4825
839 [Fig. 12]

Next, we subtract 3 from the 2; however, because this is not possible, instead, we subtract it from

the 82, where we receive 79. Hence, this yields the following figure…
Abdallah 19

4795
839 [Fig. 13]

Now, we subtract the 9 from the 5; however, because this is also not possible, will subtract it

from 95 instead, where we receive 86. This will leave us with 4,786, our final solution.

Example 3: Halve 5,625.

This problem will be solved using base 60, just as the Babylonians solved many of their
1
problems. The first step is to halve the 5 in the ones place, where we get 2 . We put the 2 in
2

1 1
place of the 5 in the ones place of 5,625 and we place the under. We will write 30 instead of 2
2

because we are using base 60. This yields to the following figure.

5622
30 [Fig. 14]

Next, we halve the 2 in the tens place, where we receive 1 and replace that 2 with the 1. We also

halve the 6, where we receive 3 and replace that 6 with the 3, as shown in figure 15.

5312
30 [Fig. 15]

Last, we halve the 5 in the thousands place. We actually halve 50 and receive 25. We place the 2

in place of the 5 and add the 5 from 25 to the 3. This yields our final solution, shown in figure

16.

2812
30 [Fig. 16]

Example 4: Multiply 325 by 243.

We write this as follows…


Abdallah 20

325
243 [Fig.17]

The first step is to multiply the 3 of the multiplicand by the 2 of the multiplier which gives us 6.

We write this as shown in the following figure.

6 325
243 [Fig. 18]

If the product was other than 6 and contained a number in the tens place value, we would have

put the number in the ones place on top of the 2 (same position as it is now) and the number in

the tens place to the left of it.

Next, we multiply the 3 of the multiplicand by the 4 of the multiplier which gives us 12. We add

the ones from the tens place in 12 to the 6, which gives us 7 and put the 2 to the right of it as

shown in figure 19.

72325
243 [Fig. 19]

Now we multiply the 3 of the multiplicand by the 3 of the multiplier to give us 9. We replace the

3 of the multiplicand with this 9 and we shift the multiplier one place to the right, as shown in

figure 20.

72925
243 [Fig. 20]

Next, we multiply the 2 of the multiplicand (in the tens place) by the 2 in the multiplier to get 4.

We add this to the 2 in the multiplicand and get 6. Then, we multiply the 2 in the multiplicand

with the 4 in the multiplier and get 8. We add this to the 9 in the multiplicand. Last, we multiply

the 2 in the multiplicand with the 3 in the multiplier, where we get 6. We place this 6 in place of
Abdallah 21

the 2 in the multiplicand. We then shift the numbers in the multiplier one place to the right, as

shown in figure 21.

77765
243 [Fig. 21]

Our final step is to multiply the 5 in the multiplicand by all of the numbers in the multiplier.

First, we multiply it by the 2, which gives us 10. We place add the 1 to the 7 in the multiplicand

in the thousands place, as shown in figure 6. Then we multiply the 5 by the 4 and receive 20. We

add the 2 to the 7 in the multiplicand in the hundreds place, as shown in figure 7. Last we

multiply the 5 by the 3 and receive 15. We add the 1 to the 6 in the multiplicand in the tens place

and the 5 replaces the 5 in the ones place, where we receive our final solution, as shown in figure

24.

78765
243 [Fig. 22]

78965
243 [Fig. 23]

78975
243 [Fig. 24]

Khayyam
Abdallah 22

Omar Khayyam was born in Persia in 1048 AD and died in 1131 AD. He was a well-

known Persian mathematician, astronomer, philosopher, and poet. Khayyam was well-known for

his work in geometry, notably his work on proportions. He completed the algebra treaty, titled

“Treatise on Demonstration of Problems on Algebra”. In these treatises he discusses the solution

of cubic equations by intersecting conic sections; he intersects a hyperbola with a circle to obtain

an answer for a cubic equation. These treatises are considered the first treatment of parallel

axioms which is based mostly on intuitive postulates.

Khayyam on the Reform of the Persian Calendar

Khayyam was a part of a panel that introduced several modifications to the Persian

calendar; these modifications were accepted as the official calendar of Persia. The Seljuk Sultan

Sultan Jalal al-Din Malekshah Saljuqi invited Khayyam to reform the Persian calendar in 1073.

Accompanied by other admired scientist, the calendar was completed in 1079, based on

Khayyam and other scientists calculations and was known as the Jalili Calendar. The calendar

included 2,820 solar years and 1,029,983 days. The Jalili calendar is agreed to be more accurate

than the Gregorian calendar because it is based on solar transit, which is the movement of any

object passing between the sun and the earth. It also requires an Ephemeris, which is a book that

provides the calculated position of celestial objects at intervals throughout a period of time. The

Jalili calendar had an error of one day in 3,770 years, whereas the Gregorian calendar has an

error of one day for every 3,330 years. Khayyam measured the length of a year as

365.24219858156 days. He rounds his results to the nearest eleventh decimal place; it is clear to

see the high level of accuracy Khayyam had.


Abdallah 23

The Persian calendar is made up of 12 months and they are: Farvardin (31 days),

Ordibehesht (31 days), Khordad (31 days), Tir (31 days), Mordad (31 days), Shahrivar (31 days),

Mehr (30 days), Aban (30 days), Azar (30 days), Day (30 days), Bahman (30 days), Esfand (29

days in an ordinary year and 30 days in a leap year). The first year begins at vernal equinox,

which is when the sun is exactly above the equator and the northern hemisphere starts to tilt

towards the sun. If the vernal equinox falls before noon on a particular day, then that day is

considered the first day and if it falls after noon, then the next day is considered the first day of

the year.

Similarly to the Islamic calendar, years are counted beginning from Muhammad’s (peace

be upon him) emigration to Medina which took place in AD 622. The Persian calendar also

includes leap years, which occurs when there are 366 days between two Persian New Year’s

days. Because the Persian calendar is based on the vernal equinox, there remain constraints on

adjusting the beginning of the calendar to the beginning of the day (midnight). Therefore, the

Persian calendar runs short of the tropical year by about 5h, 48m, 45.2s each year. Further, the

length of a year shortens by 0.00000615th of a day each century. To make up for these losses

leaps years are included mostly every 4 years. Four-year leap years add one-fourth of a day, or

0.25, to each year in the period. However, this is more than what is lost and therefore, there is

overcompensation. To overcome this, after every 6 to 7 four-year leap years, there is a five-year

leap year, which means the nest leap year occurs after 4 normal years instead of 3.

Application of Mathematics
Abdallah 24

The Muslims applied the knowledge they gained in mathematics throughout their daily

lives. Next, we will look at a few different ways math was used to help people with the

calculation of inheritance, Zakat (charity), and with creating art.

Inheritance: The Prophet Muhammad, (peace be upon him), said, “Learn the laws of inheritance

and teach them to people, for that is half of knowledge”. [23] In Islam, when a person dies, there

are specific requirements on the laws of inheritance. The arithmetic of fractions can be used to

solve the calculation of the legal shares of a person who dies and leaves no legacy of the natural

heir. We will look at two examples from Al-Khwarizmi’s work to illustrate the arithmetic.

Example 1: “A women dies, leaving her husband, a son, and three daughters, and the object is to

calculate the fraction of her estate that each heir will receive.” [9]
1
Solution: The Islamic law states that, in this case, the husband receives of the estate and that
4

the son receives double the amount the daughter receives. (It should be noted that the son or

husband is responsible for the financial well being of their sister or wife, hence.). After the
3
husband takes his share, the remainder of the estate, is then divided into five parts: two for the
4

son and three for the daughters. The least common multiple of five and four is twenty; therefore

the estate should be divided into twenty equal parts. Of these, the husband gets five, the son

receives six, and each daughter receives three.

Example 2: “A women dies, leaving her husband, son, and three daughters, but she also
1 1
bequeaths to a stranger 8 + 7 of her estate. Calculate the shares of each.”[9] (As a side note, “the
Abdallah 25

law on legacies states that a legacy cannot exceed one-third of the estate unless the natural heirs

agree to it.”)
1 1 1
Solution: Since8 + 7 ≤ 3, no complications occur and we can move forward with the calculation.

The least common denominator of the legal shares is 20. After the stranger’s legacy is paid,

1 1 15 41
which is calculated by adding 8 + 7 , this gives us56, we have remaining. The ratio of the
56

strangers share to the total share of the family is 15: 41. Now we will multiply both numbers by

20, the least common denominator, to compute of the shares of the inheritors. We have 20 ×

(15 + 41) = 20 × 56 = 1120. The stranger receives 20 × 15 = 300 and the family receives

20 × 41 = 820. The husband receives one-fourth of 820, which is 205; the son receives six-

twentieths, which is 246; and each daughter receives the remaining, which would yield 123 for

each.

Conclusion

Muslim mathematicians have contributed a great deal of knowledge to the development

of mathematics. They have expanded on the mathematical work of other great scholars and have

also developed their own mathematical work and ideas. Without their dedication, we may not

know some of the information we use to this day.

From Al-Khwarizmi, we are able to learn how he solves different types of quadratic

equations, algebraically and geometrically. From Abu Kamil, we learn about how he uses false

position to solve equations, as well as using the distribution property by looking at his geometric

proof. We also have Uqlidisi, where we learn how he multiplied mixed numbers. From looking

at Kushyar’s work, we are able to see how the fundamental operations (adding, subtracting,
Abdallah 26

multiplying and dividing) were computed. Lastly, we have Khayyam and his significant

contribution to the Persian calendar.

In conclusion, it is clear to see what a great contribution these mathematicians had in the

development of mathematics. From there work, we are able to gain an insight on how they

solved mathematical problems.


Abdallah 27

References
[1] Abu Ja'far Muhammad ibn Musa Al-Khwarizmi. (n.d.). Retrieved March 26, 2015, from
http://www-gap.dcs.st-and.ac.uk/history/Mathematicians/Al-Khwarizmi.html
[2] Abu Ja'far Muhammad ibn Musa Al-Khwarizmi. (n.d.). Retrieved March 26, 2015, from
http://www-history.mcs.st-and.ac.uk/Biographies/Al-Khwarizmi.html
[3] Abu'l Hasan Ahmad ibn Ibrahim Al-Uqlidisi. (n.d.). Retrieved March 26, 2015, from http://www-
history.mcs.st-andrews.ac.uk/history/Biographies/Al-Uqlidisi.html
[4] Abu Kamil Shuja ibn Aslam ibn Muhammad ibn Shuja. (n.d.). Retrieved March 26, 2015, from
http://www-history.mcs.st-andrews.ac.uk/history/Biographies/Abu_Kamil.html
[5] Al-Karaji | biography - Persian mathematician and engineer. (n.d.). Retrieved March 26, 2015,
from http://www.britannica.com/EBchecked/topic/312020/al-Karaji
[6] Al-Khwarizmi | biography - Muslim mathematician. (n.d.). Retrieved March 26, 2015, from
http://www.britannica.com/EBchecked/topic/317171/al-Khwarizmi
[7] "Al-Karajī (or Al-Karkh http://www.encyclopedia.com/doc/1G2-2830902256.html. (n.d.). Al-
Karajī (or Al-Karkhhttp://wwwencyclopediacom. Retrieved March 26, 2015, from
http://www.encyclopedia.com/doc/1G2-2830902256.html

[8] Aminrazavi, M. (2011, September 6). Umar Khayyam. Retrieved June 29, 2015.
[9] Berggren, J. (1986). Episodes in the Mathematics of Medieval Islam. New York, NY: Springer-
Verlag.
[10] Encyclopedia of the History of Arabic Science. (n.d.). Retrieved March 26, 2015, from
https://books.google.com/books?id=s_yIAgAAQBAJ&pg=PT454&dq=Al-Sulami
equations&hl=en&sa=X&ei=lZ8RVcWDKJG1sASS5YC4DQ&ved=0CCIQ6AEwAQ#v=onepage&q=Al
-Sulami equations&f=false
[11] Institute of Arabic and Islamic Studies. (n.d.). Retrieved March 26, 2015, from
http://www.islamic-study.org/math.htm
[12] Katz, V. (2007). The Mathematics of Egypt, Mesopotamia, China, India, and Islam: A Sourcebook.
Princeton, New Jersey: Princeton University Press.
[13] (n.d.). Retrieved April 20, 2015, from
http://www.jstor.org/stable/2972073?seq=11#page_scan_tab_contents
[14] (n.d.). Retrieved March 26, 2015, from
http://islamsci.mcgill.ca/RASI/BEA/Ibn_Labban_BEA.html
[15] (n.d.). Retrieved March 26, 2015, from http://www-
history.mcs.standrews.ac.uk/Biographies/Kushyar.html
[16] (n.d.). Retrieved March 26, 2015, from http://www.math.ntnu.no/~hanche/blog/khayyam.pdf
[17] (n.d.). Retrieved May 3, 2015, from http://users.ox.ac.uk/~some3056/docs/DeCruz_PMP.pdf
[18] Omar Khayyam | biography - Persian poet and astronomer. (n.d.). Retrieved March 26, 2015,
from http://www.britannica.com/EBchecked/topic/428267/Omar-Khayyam
Abdallah 28

[19] Omar Khayyam. (n.d.). Retrieved March 26, 2015, from http://www.famousscientists.org/omar-
khayyam/
[20] Rashed, R. (2015). Classical mathematics from Al-Khwārizmī to Descartes (1st ed., Vol. 1). New
York, NY: Routledge.
[21] The Fountain Magazine - Issue - Muslim Contributions to Mathematics. (n.d.). Retrieved March
26, 2015, from http://www.fountainmagazine.com/Issue/detail/Muslim-Contributions-to-
Mathematics
[22] The Iranian Calendar -- from Eric Weisstein's World of Astronomy. (n.d.). Retrieved June 28,
2015.

[23] The Persian Calendar. (n.d.). Retrieved June 28, 2015.


Ancient Chinese Mathematics (a very short summary)

• Documented civilization in China from c.3000 BCE. Un-


til c.200 CE, ‘China’ refers to roughly to the area shown
in the map: north of the Yangtze and around the Yellow
rivers.
• Earliest known method of enumeration dates from the
Shang dynasty (c.1600–1046 BCE) commensurate with
the earliest known oracle bone script for Chinese char-
acters. Most information on the Shang dynsaty comes
from commentaries by later scholars, though many orig-
inal oracle bones have been excavated, particularly from
Anyang, its capital.

• During the Zhou dynasty (c.1046–256 BCE) and especially towards the end in the Warring States
period (c.475–221 BCE) a number of mathematical texts were written. Most have been lost but
much content can be ascertained from later commentaries.

• The warring states period is an excellent example of the idea that wars lead to progress. Rapid
change created pressure for new systems of thought and technology. Feudal lords employed
itinerant philosophers1 ) The use of iron in China expanded enormously, leading to major changes
in warfare2 and commerce, both of which created an increased need for mathematics. Indeed
the astronomy, the calendar and trade were the dominant drivers of Chinese mathematics for
many centuries.

• The warring states period ended in 221 BCE with the victory of victory of the Qin Emperor Shi
Huang Di: famous for commanding that books be burned, rebuilding the great walls and for
being buried with the Terracotta Army in Xi’an. China was subsequently ruled by a succession
of dynasties until the abolition of the monarchy in 1912. While this simple description might
suggest a long calm in which Chinese culture and technology could develop in comfort, in
reality the empire experienced its fair share of rebellions, schisms and reprisals. The empire of
the Qin dynasty was tiny in comparison to the extent of modern China, and its expansion did
not happen smoothly or without reversals.

• East Asia (modern day China, Korea, Japan, etc.) is geographically isolated from the rest of the
world and, in particular, from other areas of developing civilization. To the north is Siberia, to
the west the Gobi desert, southwest are the Himalaya and to the south is jungle. During the
Han dynasty (c.200 BCE–220 CE) a network of trading routes known as the silk road connected
China, India and, through Persia, Europe. Indeed part of the purpose of the Great Wall was
the protection of these trade routes. Knowledge moved more slowly than goods, and there is
very little evidence of mathematical and philosophical ideas making the journey until many
1 Confucius was one such and served for a period as an advisor to Lu, a vassal state of the Zhou. He died at the beginning

of the warring states period, around 479 BCE, but his followers turned his teachings into a major philosophical school of
thought Confucianism. Its primary emphasis was stability and unity as a counter to turmoil. The other major philosophical
system dating from this time is Taoism which is more comfortable with change and adaptation.
2 Sun Tzu’s military classic The Art of War dates from this time.
centuries later. For instance, there is no evidence of the Chinese using sexagesimal notation
to aid their calculations, which meant that essentially none of Babylonian and Greek work on
astronomy made it to China. Similarly, there are many mathematical ideas which saw no ana-
logue in the west until many centuries after Chinese mathematicians had invented them. It
seems reasonable to conclude that Chinese and Mediterranean mathematics developed essen-
tially independently.

Important Mathematical Texts

The oldest suspected3 mathematical text is the Zhou Bi Suan Jing (The Mathematical Classic of the
Zhou Gnomon4 and the Circular Paths of Heaven). It was probably compiled some time in the period
500–200 BCE. The text contains, arguably, the earliest statement of Pythagoras’ Theorem as well as
simple rules for computing fractions and conducting arithmetic. The book was largely concerned
with astronomical calculations and presented in the form of a dialogue between the 11th century BCE
Duke of Zhou5 and Shang Gao (one of his ministers, and a skilled mathematician).

As with other early texts, there is no rigorous notion


of proof or axiomatics, merely strong assertions or ‘ob-
vious’ appeals to pictures. For instance, the ‘proof’
of Pythagoras’ Theorem is purely pictorial: taking the
sides of the triangle to be a, b, c in increasing order of
length, the picture essentially claims that

1
c2 = 4 · ab + (b − a)2
2
giving the familiar result when multiplied out.
Of course the Chinese do not attribute this result to Pythagoras! Instead it is known as the gou gu
where these words refer first to the shorter and then the longer of the two non-hypotenuse sides of
the triangle.

There are several other early works; here are two of the most important.
Suanshu Shu (A Book on Arithmetic) Compiled c.300–150 BCE, covering, amongst other topics, frac-
tions, the areas of rectangular fields, and the computation of fair taxes.
Jiu Zhang Suan Shu (Nine Chapters on Mathematical Arts) Written c.300 BCE–200 CE. Many top-
ics are covered, including square roots, working with ratios (false position and the rule of three6 ),
simultaneous equations, areas/volumes, right-angled triangles, etc. The Nine Chapters is a hugely
influential text, in no small part due to the creation of a detailed commentary and solution manual to
its 246 problems written by the mathematician Liu Hui in 263 CE.

3 Ancient Chinese texts are extremely difficult to date: we have no original copies and most seem to have been compiled
over several hundred years. Most of what we know of these texts is in the form of commentaries by later authors.
4 Gnomon: “One that knows or examines” — strictly the elevated piece of a sun/moondial which would have been used

for measuring said circular paths of heaven.


5 Credited with writing the I Ching, the ‘classic of changes.’
6 Given equal ratios a : b = c : d, where a, b, c known, then d = bc .
a

2
The style of both text consists of laying out methods of solution which have wide application, rather
than on proving that a particular method is guaranteed to work. Indeed there is no notion of ax-
iomatics on which one could construct a proof in the modern sense of the word.
Liu made other contributions to mathematics, including accurate
estimates of π made similarly to Archimedes. He made particu- Ao
lar use of the out-in principle which essentially describes how to Bi
compare areas and volumes:
Co
1. Areas and volumes are invariant under translation.
2. If a figure is subdivided, the sum of the areas/volumes of Bo
the parts equals that of the whole.
Ci Ai
For instance, Liu gave the argument shown in the picture as an
alternative proof of the gao gu: the green square is subdivided
and the in pieces Ai , Bi , Ci translated to new positions Ao , Bo , Co
to assemble the required squares.
Liu extended the out-in principle to analyze solids, comparing the volumes of four basic solids:

• Cube (lifang)

• Right triangluar prism (qiandu)

• Rectangular pyramid (yangma)

• Tetrahedron (bienuan)

These could be assembled to calculate the volume of, say, a truncated pyramid:

The Bamboo Problem This problem is taken from the Nine Chapters:
indeed the picture shows the problem as depicted in Yang Hui’s
famous Analysis of the Nine Chapters from 1261.

A bamboo is 10 chi high. It breaks and the top touches the ground 3
chi from the base of the stem. What is the height of the break?

In modern language, if a, b, c is the triangle, we know that b + c = 10


and a = 3. We want b.

1 a2 91
 
The solution is b = b+c− = chi: think about why. . .
2 b+c 100

3
Chinese Enumeration
The Chinese had two parallel systems of enumeration. Both are essentially decimal.

Oracle Bone Script and Modern Numerals The earliest Chinese writing is known as oracle bone
script dates from around 1600 BCE. The numbers 1–10 were recorded with distinct symbols, with
extra symbols for 20, 100, 1000 and 10000. These were decorated to denotes various multiples. Some
examples are shown below.

The system is quite complex, given all the possibilities for decoration, and therefore more advanced
than other contemporary systems. Standard modern numerals are a direct descendant of this script:

Observe the similarity between the expressions for the first 10 digits. The second image denotes
the number 842, where second and 4th symbols represent 100’s and 10’s respectively: literally eight
hundred four ten two. No zero symbol is required as a separator: one could not confuse 205 with 250.
The system is still partly positional: the symbol for, say, 8 can mean 800 if placed correctly, but only
if followed by the symbol for 100.

Rod Numerals The second dominant form of enumeration dates from around 300 BCE and was
in very wide use by 300 CE. Numbers were denoted by patterns known as Zongs and Hengs. These
represented alternate powers of 10: Zongs denoted units, 100’s, 10000’s, etc., while Hengs were for
10’s, 1000’s, 100000’s, etc.

Rod numerals were immensely practical. In extremis they could easily be scratched in the dirt. More
commonly they were created using short bamboo sticks or counting rods, of which any merchant
worth their salt would carry a bundle. They were often used in conjunction with a counting board:
a grid of squares on which sticks could be placed for ease of calculation. This technology was in-
valuable for keeping calculations accurate and facilitating easy trade. They also made for several
calculation methods which will seem familiar. There was no need for a zero in this system, as an
empty space did the job.

4
Addition and subtraction are straightforward by carrying and borrowing in the usual way. The
smallest number was typically placed on the right. Multiplication is a little more fun: for instance to
multiply 387 by 147, one would set up the counting board as follows:
3 8 7
Arrange rods: use modern numerals for clarity
1 4 7

3 8 7
4 4 1 441 = 147 × 3: note the position of 147 under the 3
1 4 7

8 7
4 4 1
Delete 3, move 147 and multiply: 1176 = 147 × 8
1 1 7 6
1 4 7

8 7
5 5 8 6 Sum rows
1 4 7

7
5 5 8 6
Delete 8, move 147 and multiply: 1029 = 147 × 7
1 0 2 9
1 4 7

5 6 8 8 9 Sum rows: 387 × 147 = 56889


1 4 7
The algorithm is essentially long-multiplication, but starting with multiplication by the largest digit
instead of the units as we are used to.

Division essentially works like long-division: to divide 56889 by 147 one might have the following
sequence of boards
3 3 8 3 8 7
5 6 8 8 9 −→ 5 6 8 8 9 −→ 1 2 7 8 9 −→ 1 0 2 9
1 4 7 1 4 7 1 4 7 1 4 7
In the first two boards. 147 goes 3 times into 568.
In board 3, we subtract 3 × 147 from 568 to leave 127, shift the 147 over and observe that 147 goes 8
times into 1278. In the final step we have subtracted 8 × 147 from 1278 to leave 1029 before shifting
the 147 to its final position. Since 147 divides exactly seven times into 1029, we are done.
There is nothing stopping us from dividing numbers where the result is not an integer: simply con-
tinue as in long-division.

Similtaneous equations The counting board could be set up to compute solutions to simultaneous
linear equations. Essentially the coefficients of a linear system were placed in adjacent columns and

5
then column operations were performed. The method is thus identical to what you learn in a linear
algebra class, but with columns rather than rows. For instance, a linear system could be encoded
thus:
( 3 2 1 2 1 1 0 1
3x + 2y = 7
−→ 2 1 −→ 1 1 −→ 1 0 −→ 1 0 −→ x = 1, y = 2
2x + y = 4 7 4 3 4 3 1 2 1
This matrix method was essentially unique to China until the 1800’s.

Euclidean algorithm The counting board lent itself to the computation of greatest common divi-
sors, which were used very practically for simplifying fractions. Here is the process applied to 35
91 :,

35 35 35 14 14 7
91 56 21 21 7 7
At each stage, one subtracts the smaller number from the larger. Once the same number is in each
row you stop. You should recognize the division algorithm at work. . . Since gcd(35, 91) = 7, both
35 5
could be divided by 7 to obtain 91 = 13 in lowest terms.

Negative numbers There is a strong case for arguing that the Chinese are also the oldest adopters
of negative numbers. These were not thought of as numbers per se, rather different colored rods
could be used to denote a deficiency in a quantity. Indeed the Nine Chapters describes using red
rods for positives and black for negatives. This would commonly be used when adding up accounts.
This practice was known by around 1 CE, roughly 500 years before negative numbers were used in
calculations in India.

Music, Mysticism and Approximations Like the Pythagoreans, the Chinese were interested in mu-
sic and pattern for mystical reasons. While the Pythagoreans delighted in the pentagram, the Chinese
created magic squares7 as symbols of perfection. The notion of equal temprament in musical tuning
was first ‘solved’ in China by Zhu Zaiyu (1536–1611), some 30 years before Mersenne & Stevin pub-
lished the same result in Europe. This required the computation of the twelfth-root of 2 which Zhu
computed using approximations for square and cube roots:
rq

12 3 √
2= 2

Zhu’s approximation was correct to 24 decimal places!


Indeed the Chinese emphasis on practical methods meant that they often had the most accurate
approximations for their time:
√ 355 377
• Approximations to π including 227 , 10, 113 , 120 . Most accurate in the world from 400–1400 CE.

• Methods for approximating square and cube roots were found earlier than in Europe. Approxi-
mations for solutions to higher-order equations similar to the Horner–Ruffini method were also
discovered earlier.
• Pascal’s triangle first appears in China around 1100 CE. It later appeared in Islamic mathematics
before making its way to Europe.
7 Grids where all rows and columns sum to the same total.

6
Two famous problems
We finish with a discussion of two famous Chinese problems. The first is known as the Hundred Fowls
Problem and dates from the 5th century CE. It was copied later in India and then by Leonardo da Pisa
(Fibonacci) in Europe, thus showing how Chinese mathematics travelled westwards.

If cockerals cost 5 qians8 each, hens cost 3 qians each, and 3 chickens cost 1 qian, and if 100
fowls are bought for 100 qians, how many cockerels, hens and chickens are there?
In essence, the problem asks us to find non-negative integers satisfying

5x + 3y + 13 z = 100
(

x + y + z = 100

The answers are simply stated as (4, 18, 78), (8, 11, 81), (12, 4, 84) while the possible solution (0, 25, 75)
was not stated.9

Finally an example of the Chinese Remainder Theorem for solving simultaneous congruence equations.
The Theorem dates from the 4th century CE: it travelled to India where it was described by Bhra-
magupta and thence to Europe. This example comes from Qin Jiushao’s Shu Shu Jiu Zhang (Nine
Sections of Mathematics) in 1247.

Three thieves entered a rice shop and stole three identical vessels filled to the brim with
rice, but whose exact capacity was not known. The thieves were caught and their vessels
examined: all that was left in vessels X, Y and Z were 1 ge, 14 ge and 1 ge respectively. The
thieves did not know the exact quantities they’d stolen. A used a “horse ladle” (capacity
19 ge) to take rice from vessel X. B used a wooden shoe (capacity 17 ge) to take rice from
vessel Y. C used a bowl (capacity 12 ge) to take rice from Z. What was the total amount of
rice stolen?

In modern language the total amount of rice N satisfies



1 (mod 19)

N ≡ 14 (mod 17)
1 (mod 12)

The answer is that N = 3193 ge. This is the smallest possible solution: all such are congruent modulo
19 · 17 · 12 = 3876.
8A qian is a copper coin.
9 As a possible method, one substitutes z = 100 − x − y in first equation to obtain 7x + 4y = 100, or
7
y = 25 − x
4
Since y ∈ Z, we must have x = 4m for some integer m. Thus all solutions have the form

x = 4m, y = 25 − 7m, z = 75 + 3m, m∈Z

For m = 1, 2, 3 and 0 we obtain the above solutions. For m ≥ 4 we have y < 0, while m < 0 yields x < 0.

You might also like